Вы находитесь на странице: 1из 303

1.

For the reversible reaction


X

which of the following could represent the change in the concentrations of X and Y with time,
starting with a mixture of both X and Y? Equilibrium is reached at time t eqm.

(Total 1 mark)

2.

(a)

In the catalytic converter of a car engines exhaust system, the following reaction occurs.
2NO(g) + 2CO(g)

N2(g) + 2CO2(g)

H = 745 kJ mol

The temperature in a catalytic converter is high.


(i)

State the effect, if any, on the position of equilibrium if the temperature is lowered.
Give a reason for your answer.
Effect.................................................................................................................
Reason...............................................................................................................
...........................................................................................................................
(2)

Sri Lankan School

(ii)

The gases from the engine are not cooled before entering the converter. Explain
why this is so.
...........................................................................................................................
...........................................................................................................................
...........................................................................................................................
...........................................................................................................................
(2)

(iii)

State the effect, if any, on the position of equilibrium if the pressure on the reacting
gases is increased. Give a reason for your answer.
Effect................................................................................................................
Reason...............................................................................................................
...........................................................................................................................
(2)

(b)

Nitrogen monoxide, NO, is formed when nitrate ions, NO 3 , in acidic solution are
reduced by silver metal.

(i)

Calculate the oxidation number of nitrogen in NO and in NO 3

In NO...............................................................................................................

In NO 3 .............................................................................................................

(2)

(ii)

Balance the half-equation for the reduction of nitrate ions, NO 3 , in acidic solution.

NO 3 + .............H+ + ........e NO + ........H2O


(2)

(iii)

Write the half-equation for the oxidation of silver metal, Ag, to silver ions, Ag .
(1)

(iv)

Hence deduce the full ionic equation for the reaction between silver metal and
nitrate ions in acidic solution. State symbols are not required.
(2)
(Total 13 marks)

Sri Lankan School

Sri Lankan School

3.

In which reaction is water acting as a Brnsted-Lowry acid?


+

H2O + HCl H3O + Cl

H2O + SO3 H2SO4

H2O + NH3 NH4 + OH

H2O + CO2 H2CO3

(Total 1 mark)

4.

Which of the following solutions has the lowest pH?


3

0.010 mol dm hydrochloric acid.

0.100 mol dm hydrochloric acid.

0.010 mol dm ethanoic acid.

0.100 mol dm ethanoic acid.

3
3
3

(Total 1 mark)

5.

Which of the following solutions, when mixed, would make a buffer with pH more than 7?
A

Methanoic acid and sodium methanoate.

Sodium hydroxide and sodium chloride.

Ammonia and ammonium chloride.

Ammonium chloride and ammonium ethanoate.


(Total 1 mark)

Sri Lankan School

6.

This question is about the equilibrium reaction


N2(g) + 3H2(g)

2NH3(g) H = 92 kJ mol

Which statement is not correct?


2

The units of Kp are atm .

Kp increases as temperature is decreased.

Kp increases when the pressure increases.

Kp increases when the total entropy change, Stotal, increases.


(Total 1 mark)

7.

1,2-dichloroethane decomposes in the presence of a catalyst.


CH2ClCH2Cl(g)

CH2=CHCl(g) + HCl(g)

H = +51 kJ mol

Which of the following would result in an increase in the equilibrium yield of chloroethene?
A

Increasing the temperature.

Increasing the pressure.

Increasing the surface area of the catalyst.

Changing the catalyst to a more efficient one.


(Total 1 mark)

Sri Lankan School

8.

Ethanoic acid can be manufactured by the following reaction, which is carried out between
150 C and 200 C.
CH3OH(g) + CO(g)
(a)

CH3COOH(g)

A mixture of 50.0 mol of methanol and 50.0 mol of carbon monoxide reaches equilibrium
at a pressure of 32.0 atm. At 175 C, the equilibrium partial pressure of ethanoic acid is
22.2 atm.
(i)

Write the expression for the equilibrium constant in terms of pressure, K p, for this
reaction.
(1)

(ii)

Calculate the partial pressures of methanol and carbon monoxide at equilibrium.


Methanol...............................................................................................
Carbon monoxide................................................................................
(2)

(iii)

Calculate the value of Kp for this reaction at 175 C. Include a unit in your answer
and give your answer to three significant figures.
(2)

(b)

Another sample of 50.0 mol of methanol and 50.0 mol of carbon monoxide was allowed
to reach equilibrium at the same pressure of 32.0 atm, but at a lower temperature. 93.6%
of the methanol was converted at equilibrium.
(i)

Complete the table below to show the number of moles of each species in the
equilibrium mixture.

CH3OH
Number of moles at
start

50.0

CO

CH3COOH

50.0

Number of moles at
equilibrium
(2)

(ii)

Calculate the partial pressure of ethanoic acid in the equilibrium mixture.


(1)

(iii)

Is the reaction exothermic or endothermic? Explain your answer.


...........................................................................................................................
...........................................................................................................................
...........................................................................................................................
...........................................................................................................................

Sri Lankan School

(1)

Sri Lankan School

(c)

How, if at all, does the addition of methanol to the equilibrium mixture affect the
following? Justify your answers.
CH3OH(g) + CO(g)
(i)

CH3COOH(g)

The equilibrium constant for the formation of ethanoic acid.


...........................................................................................................................
...........................................................................................................................
(1)

(ii)

The equilibrium yield of ethanoic acid.


...........................................................................................................................
...........................................................................................................................
...........................................................................................................................
(1)

(d)

In industry, catalysts are used even though they are often expensive.
State and explain ONE benefit to the environment resulting from the use of catalysts in
industrial processes.
.....................................................................................................................................
.....................................................................................................................................
.....................................................................................................................................
.....................................................................................................................................
(2)
(Total 13 marks)

Sri Lankan School

9.

Vinegar is used as a food preservative. It is an acidic solution containing ethanoic acid,


CH3COOH.
(a)

A titration was carried out to measure the concentration of ethanoic acid in a sample of
3
vinegar. 25.0 cm of a vinegar solution was titrated with a solution of sodium hydroxide,
3
concentration 0.250 mol dm . The concentration of the ethanoic acid in the vinegar
3
solution was found to be 0.125 mol dm .
(i)

Calculate the pH of 0.250 mol dm sodium hydroxide at 298 K.


[Kw = 1.00 10

14

mol dm at 298 K.]


(2)

(ii)

Write the expression for the acid dissociation constant, Ka, for ethanoic acid.
(1)

(iii)

Calculate the pH of 0.125 mol dm ethanoic acid at 298 K.


5

[Ka for ethanoic acid is 1.7 10 mol dm at 298 K.]


(2)

Sri Lankan School

(iv)

When half the ethanoic acid is neutralized, the concentration of the remaining
ethanoic acid equals the concentration of the sodium ethanoate which has formed.
What is the pH of the mixture at this point? Justify your answer.
pH.....................................................................................................................
Justification.......................................................................................................
...........................................................................................................................
(2)

(v)

On the axes below, sketch the titration curve for this reaction when 30 cm of the
3
sodium hydroxide is added to 25.0 cm of the vinegar solution.

(3)

Sri Lankan School

10

(vi)

The only indicators which were available for this titration were methyl yellow (in
ethanol) and thymolphthalein. Explain which indicator is more suitable for this
titration and why the other is unsuitable. You will need to refer to your data
booklet.
...........................................................................................................................
...........................................................................................................................
...........................................................................................................................
...........................................................................................................................
...........................................................................................................................
(2)

(b)

In the food industry, ethanoic acid is described as an acidity regulator, additive number
E260.
Ethanoic acid can neutralize alkalis. What substance could be mixed with ethanoic acid
so that it regulates pH as a buffer in foodstuffs?
.....................................................................................................................................
(1)
(Total 13 marks)

10.

The electrode potential for a cell can be used to calculate the equilibrium constant for the cell
reaction. This is because

ln E

In E

cell is proportional to lnK

cell

is proportional to K.

cell

cell

is proportional to lnK.
is proportional to K.
(Total 1 mark)

Sri Lankan School

11

11.

Magnesium nitrate, Mg(NO3)2, decomposes when it is heated. One product is the brown gas,
nitrogen dioxide.
(a)

(i)

Write an equation for this reaction. State symbols are not required.
(2)

(ii)

Calcium nitrate decomposes in a similar way to magnesium nitrate, but at a higher


temperature.
Explain why the two nitrates have different stability to heat.
.............................................................................................................
.............................................................................................................
.............................................................................................................
.............................................................................................................
.............................................................................................................
.............................................................................................................
(2)

(b)

Sodium nitrate decomposes to give different products to magnesium nitrate. Write an


equation for the decomposition of sodium nitrate. State symbols are not required.
(1)

(c)

A student suggested that the structure of the nitrate ion, NO 3 , is

Scientists have found that the bonds between nitrogen and oxygen in the nitrate ion are all
the same length. Is the students suggestion supported by this evidence? Explain your
answer.
.....................................................................................................................................
.....................................................................................................................................
.....................................................................................................................................
(1)

Sri Lankan School

12

(d)

Nitrogen dioxide gas can dimerize to dinitrogen tetroxide, N2O4, a very pale yellow gas,
as shown in the equation below.
2NO2(g)
(i)

N2O4(g)

H = 58 kJ mol

What would you see when an equilibrium mixture of these gases is warmed gently?
Explain your answer.
...........................................................................................................................
...........................................................................................................................
...........................................................................................................................
...........................................................................................................................
(2)

(ii)

Explain why an equilibrium mixture of these gases eventually becomes paler in


colour when the pressure on it is increased.
...........................................................................................................................
...........................................................................................................................
...........................................................................................................................
...........................................................................................................................
(2)

Sri Lankan School

13

(e)

Two Maxwell-Boltzmann distributions showing the energy of particles in a gas at


different temperatures, T1 and T2, are shown below. The activation energy for the reaction
is labelled EA.

Use the distributions to explain why gases react faster when the temperature is increased.
.....................................................................................................................................
.....................................................................................................................................
.....................................................................................................................................
.....................................................................................................................................
.....................................................................................................................................
.....................................................................................................................................
.....................................................................................................................................
(2)
(Total 12 marks)

Sri Lankan School

14

12.

Consider the equilibrium


Cl2(g) + PCl3(g)

PCl5(g)

Which of the following is true when the total pressure of the system is increased at constant
temperature?

Value of Kp

Mole fraction of PCl5(g)

decreases

decreases

unaltered

increases

decreases

increases

unaltered

unaltered
(Total 1 mark)

13.

In which of the following reactions is nitric acid acting as a base?


A

HNO3 + NaOH NaNO3 + H2O

HNO3 + H2O H3O+ + NO3

HNO3 + H2SO4 H2NO3 + HSO4

HNO3 + NaHCO3 NaNO3 + H2O + CO2

(Total 1 mark)

14.

Why does phenolphthalein, which is colourless in acidic solutions, turn pink in alkaline
solutions?
A

It is oxidized to a pink compound by hydroxide ions.

It forms a pink anion by loss of H ions.

It forms a pink anion by gain of H ions.

It forms a pink cation by gain of H ions.

(Total 1 mark)

Sri Lankan School

15

15.

The dissociation of ethanoic acid in aqueous solution is represented by


CH3COOH(aq) + H2O(l)

H3O (aq) + CH3COO (aq)

Which of the following statements is true for this equilibrium?

CH3COOH is an acid and its conjugate base is CH3COO .

H2O is an acid and its conjugate base is OH .

At equilibrium, the concentrations of each substance are the same.

At equilibrium, the reaction from left to right and the reaction from right to left have
stopped.

(Total 1 mark)

16.

Why are aqueous solutions of sodium ethanoate slightly alkaline?


A

The sodium ions react with water to give an alkali.

The ethanoate ions react with water to give hydroxide ions.

All sodium salts give alkaline solutions.

The sodium ethanoate is fully ionized in solution.


(Total 1 mark)

17.

A solution of a weak acid cannot be titrated with a weak base using an indicator to find the endpoint because
A

the pH change is too gradual close to the equivalence point.

there are too few H ions to affect the indicator.

there are too few OH ions to affect the indicator.

the pH change occurs outside the range of any indicator.

(Total 1 mark)

Sri Lankan School

16

18.

Iron and steam at high temperature react in a closed vessel to give an equilibrium mixture
3Fe(s) + 4H2O(g)

Fe3O4(s) + 4H2(g)

Which of the following is the correct expression for Kp?

Kp
A
Kp
B

Kp
C
D

PH 2
PH 2O
PFe 3 O 4 PH42
PFe3 PH42O

PH42
PH42O

K p PH42
(Total 1 mark)

19.

At 100 C, pure water has a pH of 6, whereas at 25 C it has a pH of 7. This is because


A

the dissociation of water is endothermic, so the concentration of hydrogen ions is lower at


100 C than it is at 25 C.

the dissociation of water is exothermic, so the concentration of hydrogen ions is lower at


100 C than it is at 25 C.

the dissociation of water is endothermic, so the concentration of hydrogen ions is higher


at 100 C than it is at 25 C.

at 100 C, water has a higher concentration of hydrogen ions than of hydroxide ions.
(Total 1 mark)

20.

(a)

(i)

Define pH.
.............................................................................................................
(1)

(ii)

Calculate the pH of 0.0100 mol dm hydrochloric acid, which is a strong acid.


(1)

Sri Lankan School

17

Sri Lankan School

18

(b)

Ethanoic acid is a weak acid with an acid dissociation constant, K a, of value 1.75 10

mol dm at 25 C.
(i)

Calculate the pH of 0.0100 mol dm ethanoic acid at 25 C, stating any ONE


assumption that you have made.
(4)

(ii)

The pH of hydrochloric and of ethanoic acid at two different concentrations is


given in the table.

pH of 0.00100 mol dm
solution

pH of 0.000100 mol dm
solution

Hydrochloric acid

3.0

4.0

Ethanoic acid

3.9

4.4

In the case of hydrochloric acid, dilution by a factor of 10 increases the pH by one


unit. Suggest why ethanoic acid behaves differently.
...........................................................................................................................
...........................................................................................................................
...........................................................................................................................
...........................................................................................................................
...........................................................................................................................
...........................................................................................................................
...........................................................................................................................
...........................................................................................................................
(2)

Sri Lankan School

19

(c)

Orange marmalade usually contains sodium citrate as a preservative. Together with the
fruit in the marmalade, it forms a buffer solution which, at a suitable pH, inhibits mould
growth.
(i)

Define the term buffer solution.


...........................................................................................................................
...........................................................................................................................
...........................................................................................................................
(2)

(ii)

What is the substance in the fruit that produces a buffer with sodium citrate?
...........................................................................................................................
(1)

(iii)

Explain how a buffer solution works using this system or any other of your choice.
Support your explanation with equations.
...........................................................................................................................
...........................................................................................................................
...........................................................................................................................
...........................................................................................................................
...........................................................................................................................
...........................................................................................................................
...........................................................................................................................
...........................................................................................................................
...........................................................................................................................
...........................................................................................................................
...........................................................................................................................
...........................................................................................................................
...........................................................................................................................
...........................................................................................................................
(4)
(Total 15 marks)

Sri Lankan School

20

Sri Lankan School

21

21.

Hydrogen is used in very large quantities as a fuel, as a reducing agent, and in the production of
ammonia. Hydrogen is manufactured by steam reforming of methane from natural gas. Two
reactions are involved, both being in equilibrium in closed systems.

(a)

Reaction I

CH4(g) + H2O(g)

Reaction II

CO(g) + H2O(g)

CO(g) + 3H2(g) H = + 210 kJ mol


CO2(g) + H2(g)

H = 42 kJ mol

Write the expression for the equilibrium constant, Kp, for reaction I.
(1)

(b)

Reaction I occurs at a temperature of 1000 K and a pressure of 30 atm over a nickel


catalyst.
(i)

State and explain the effect, if any, on the value of Kp of increasing the pressure on
the reaction.
...........................................................................................................................
...........................................................................................................................
...........................................................................................................................
(1)

(ii)

Explain, in terms of your answers to (a) and (b)(i), why an increase in the pressure
leads to a decrease in yield in reaction I.
...........................................................................................................................
...........................................................................................................................
...........................................................................................................................
...........................................................................................................................
...........................................................................................................................
(2)

Sri Lankan School

22

(iii)

Increasing the pressure on this heterogeneously-catalysed reaction I has very little


effect on the rate of the reaction. Suggest why this is so.
...........................................................................................................................
...........................................................................................................................
...........................................................................................................................
...........................................................................................................................
...........................................................................................................................
(2)

(c)

The expression for Kp for reaction II is


Kp

PCO 2 PH 2
PCO PH 2 O

At a particular temperature and 30 atm pressure, a mixture of equal amounts of carbon


monoxide and steam react to give an equilibrium mixture where 75% of the CO has
reacted.
Calculate the value of Kp showing your working.
(3)

(d)

Carbon dioxide and hydrogen are separated by washing the gas with potassium carbonate
solution to give potassium hydrogencarbonate solution, leaving hydrogen in the gas
stream. Potassium carbonate is expensive and is regenerated by heating the potassium
hydrogencarbonate and liberating the carbon dioxide.
(i)

Hydrogen is often claimed to be a non-polluting fuel as it only produces water on


burning.
Explain why its manufacture using reactions I and II does not support this claim.
...........................................................................................................................
...........................................................................................................................
...........................................................................................................................
(1)

Sri Lankan School

23

(ii)

Write the equation for the thermal decomposition of potassium hydrogencarbonate.


State symbols are not required.
(1)

(e)

Although industrial processes are often discussed in terms of equilibria, they are rarely
allowed to reach equilibrium.
Suggest why, apart from insufficient reaction time, this is so.
.....................................................................................................................................
.....................................................................................................................................
(1)
(Total 12 marks)

22.

In the equilibrium below, what effect would the changes described have on the system?
2H2S(g) + SO2(g)
(a)

3S(s) + 2H2O(g)

H is negative

Increase in temperature
A

increase rate, decrease yield

increase rate, increase yield

decrease rate, decrease yield

decrease rate, increase yield


(1)

(b)

Decrease in pressure
A

increase rate, decrease yield

increase rate, increase yield

decrease rate, decrease yield

decrease rate, increase yield


(1)
(Total 2 marks)

Sri Lankan School

24

23.

Iodine can react with sodium hydroxide solution to form NaIO3(aq), according to the equation
below.
3I2(aq) + 6NaOH(aq) 5NaI(aq) + NaIO3(aq) + 3H2O(l)
Which of the statements about the reaction is false?
A

The oxidation number of some iodine atoms goes up.

At high temperatures NaIO(aq) also forms.

Sodium ions are spectator ions.

The oxidation number of some iodine atoms goes down.


(Total 1 mark)

24.

2+

In the reaction between Ag (aq) ions and Fe (aq) ions, what would be the effect of increasing
+
the concentration of Ag (aq) ions?
+

2+

3+

Ag (aq) + Fe (aq)

Fe (aq) + Ag(s)
3+

Rate of reaction increases, yield of Fe (aq) stays the same.

Rate of reaction increases, yield of Fe (aq) decreases.

Rate of reaction decreases, yield of Fe (aq) stays the same.

Rate of reaction increases, yield of Fe (aq) increases.

3+

3+

3+

(Total 1 mark)

25.

Which one of these reactions is not a disproportionation reaction?


A

2H2O2(aq) O2(g) + 2H2O(l)

S2O3 (aq) + 2H (aq) SO2(g) + S(s) + H2O(l)

Cl2(aq) + 2Br (aq) 2Cl (aq) + Br2(aq)

2Cu (aq) Cu(s) + Cu (aq)

2+

(Total 1 mark)

Sri Lankan School

25

Sri Lankan School

26

26.

Almost two thirds of the worlds ethanoic acid is made using the following equilibrium reaction,
with the aid of an iridium complex as a catalyst.
CH3OH(l) + CO(g)

CH3COOH(l)

DH = 135 kJ mol

Which of the following changes in conditions would increase the equilibrium yield of ethanoic
acid?
A

increase pressure

decrease pressure

increase temperature

add a catalyst
(Total 1 mark)

27.

For the equilibrium,


N2(g) + 3H2(g)

2NH3(g)

Which is the correct expression for Kp?

[ NH 3 (g)] 2
A

[ N 2 (g)][H 2 (g)] 3
PN 2 ( g) PH 2 ( g)

PNH 3 ( g)
P 2 NH 3 ( g)

PN 2 ( g) P 3 H 2 ( g)
PN 2 ( g) P 3 H 2 ( g)

P 2 NH 3 ( g)
(Total 1 mark)

Sri Lankan School

27

28.

The expression for Kc for the equilibrium 2SO2(g) + O2(g)

2SO3(g) is

[SO 3 (g)] 2
2
Kc = [SO 2 (g)] [O 2 (g)]

What are the units of Kc in this equilibrium expression?


3

mol dm

mol dm

dm mol

atm

6
1

(Total 1 mark)

29.

For the equilibrium


2NO2(g)

N2O4(g)

DH = 57.2 kJ mol

which one of the following changes would result in a different value of the equilibrium
constant?
A

an increase in temperature

a decrease in pressure

an increase in pressure

an increase in the concentration of NO2(g)


(Total 1 mark)

Sri Lankan School

28

30.

Solutions of concentration 0.1 mol dm of iron(II) ions and silver(I) ions were mixed at room
temperature and allowed to reach equilibrium.
2+

Fe (aq) + Ag (aq)

3+

Fe (aq) + Ag(s)

Which one of the following statements is true?


A

as the equilibrium position was approached, the forward reaction became slower until it
stopped.

at the equilibrium position, no more Ag(s) reacted with Fe (aq).

at the equilibrium position, the rate of the forward reaction equalled the rate of the
backward reaction.

no Fe (aq) reacted with Ag(s) until the equilibrium position was reached.

3+

3+

(Total 1 mark)

31.

This question concerns four solutions, A to D. They were prepared by mixing equal volumes of
3
0.2 mol dm solutions of two different substances. The substances were
A

HCl(aq) and NaOH(aq)

HCl(aq) and NaCl(aq)

NH3(aq) and NH4Cl(aq)

CH3COOH(aq) and CH3CO2Na(aq)

Select, from A to D, the mixture which would:


(a)

have the lowest concentration of hydrogen ions


A
B
C
D
(1)

Sri Lankan School

29

(b)

act as a buffer of pH about 5


A
B
C
D
(1)

(c)

have a chloride ion concentration of 0.2 mol dm .


A
B
C
D
(1)
(Total 3 marks)

32.

This question concerns the titration of a solution of sodium hydroxide with a solution of
hydrochloric acid. As the titration proceeds the pH of the mixture changes.
(a)

What was the pH when 24.95 cm of 1.00 mol dm NaOH(aq) had been added to 25 cm
3
of 1.00 mol dm HCl(aq)?
A

11

(1)

Sri Lankan School

30

(b)

What was the pH when 25.05 cm of 1.00 mol dm NaOH(aq) had been added to 25 cm
3
of 1.00 mol dm HCl(aq)?
A

11

(1)

(c)

Which one of the following indicators would be most suitable to use to determine the end
point of this titration?
pH range
A

methyl violet

01.6

universal indicator

311

thymolphthalein

8.310.6

alizarin yellow R

10.113.0
(1)
(Total 3 marks)

33.

This question is about the pineapple flavouring used in sweets. It is an ester with the formula
C3H7COOCH3, which can be broken down into butanoic acid and methanol when mixed with
hydrochloric acid.
The following equilibrium is set up:
C3H7COOCH3(l) + H2O(l)
(a)

C3H7COOH(l) + CH3OH(l)

Give the name of this ester.


.....................................................................................................................................
(1)

Sri Lankan School

31

(b)

Why does the ester have a comparatively low boiling point compared to the other three
substances in the equation?
.....................................................................................................................................
.....................................................................................................................................
.....................................................................................................................................
(1)

(c)

What is the name given to this type of reaction?


.....................................................................................................................................
(1)

(d)

Suggest the reasons why manufacturers choose to use the chemically manufactured
pineapple flavouring rather than the natural product and why consumers might prefer to
choose the natural product.
.....................................................................................................................................
.....................................................................................................................................
.....................................................................................................................................
.....................................................................................................................................
.....................................................................................................................................
.....................................................................................................................................
.....................................................................................................................................
.....................................................................................................................................
.....................................................................................................................................
.....................................................................................................................................
.....................................................................................................................................
.....................................................................................................................................
.....................................................................................................................................
.....................................................................................................................................
(4)

Sri Lankan School

32

(e)

In an experiment, 10.2 g (0.10 mol) of the ester was mixed with 18 cm of 1.0 mol dm
hydrochloric acid and left until equilibrium had been reached. The hydrochloric acid acts
as a catalyst and contains 18 g (1 mol) of water. At equilibrium, 4.4 g of butanoic acid
was found to be present.
3

Molar mass of butanoic acid = 88 g; assume the total volume at equilibrium is 30 cm .


Give the expression for the equilibrium constant, Kc, for this equilibrium and calculate its
value. Explain why it has no units.

.....................................................................................................................................
.....................................................................................................................................
.....................................................................................................................................
(5)
(Total 12 marks)

34.

Methane reacts with steam in an endothermic reaction.


CH4(g) + H2O(g)

(a)

CO(g) + 3H2(g)

State the effect on the value of the equilibrium constant of an increase in temperature.
.....................................................................................................................................
.....................................................................................................................................
(1)

Sri Lankan School

33

(b)

Use your answer to (a) to explain the effect of this change on the position of equilibrium.
.....................................................................................................................................
.....................................................................................................................................
.....................................................................................................................................
.....................................................................................................................................
(2)
(Total 3 marks)

35.

This question is about the reaction of magnesium with hydrochloric acid which takes place
rapidly at room temperature.
Mg(s) + 2HCl(aq) MgCl2(aq) + H2(g)

(a)

DH = 467 kJ mol

Rewrite the equation omitting spectator ions.


(1)

Sri Lankan School

34

(b)

Suggest the sign of the following entropy changes for this reaction. Justify each of your
answers.
(i)

DSsystem
...........................................................................................................................
...........................................................................................................................
...........................................................................................................................
(2)

(ii)

DSsurroundings
...........................................................................................................................
...........................................................................................................................
...........................................................................................................................
(2)

(iii)

DStotal
...........................................................................................................................
...........................................................................................................................
...........................................................................................................................
(1)

(c)

A student carried out this experiment at five different temperatures in order to calculate
the activation energy of the reaction. The students laboratory record is shown below.
Method
3

Clean a strip of magnesium weighing 0.100 g with sand paper. Measure the temperature of 20 cm of 1.00 mol
dm

hydrochloric acid in a 100 cm beaker. Add the magnesium ribbon, stir continuously, and time how long it
takes for the magnesium to disappear. Repeat the experiment at four other temperatures.
Assumption: the initial rate of reaction is proportional to 1/time.
Results
1/T

time
/s

1/time

45

0.0222

3.81

25

0.0400

3.22

11

0.0909

-2.40

3.04 10

0.1667

-1.79

3.53 10

122

0.0082

-4.80

Temperature
/C

Temperature
/K

/K

24

297

3.37 10

33

306

3.27 10

45

318

3.14 10

56

329

10

283

Sri Lankan School

/s

ln 1/time

35

Sri Lankan School

36

The Arrhenius equation is ln k = Ea/R (1/T) + constant


ln 1/time is proportional to ln k and so a graph of ln 1/time will have the same gradient as
that of the Arrhenius plot of ln k against 1/Temperature
The student plotted the graph of ln 1/time against 1/Temperature and from this the
1
activation energy, EA, was calculated as + 51.3 kJ mol .
(i)

Suggest the reason for cleaning the magnesium ribbon with sand paper.
...........................................................................................................................
...........................................................................................................................
...........................................................................................................................
(1)

Sri Lankan School

37

(ii)

Calculate the number of moles of hydrochloric acid used up when all the
magnesium reacts in one experiment. Hence comment on whether the change in
concentration during the reaction will have a significant effect on the validity of the
assumption that the initial rate of reaction is proportional to 1/time. How would
you overcome this potential error?
[Take the relative atomic mass of magnesium as 24 in this and subsequent
calculations.]
...........................................................................................................................
...........................................................................................................................
...........................................................................................................................
...........................................................................................................................
...........................................................................................................................
...........................................................................................................................
...........................................................................................................................
...........................................................................................................................
...........................................................................................................................
...........................................................................................................................
...........................................................................................................................
...........................................................................................................................
...........................................................................................................................
...........................................................................................................................
...........................................................................................................................
...........................................................................................................................
(5)

Sri Lankan School

38

(iii)

Use the value of DH and other information given in the question to calculate the
temperature change in an experiment assuming no energy is lost to the
surroundings. Hence comment on whether this change in temperature will have a
significant effect. How would you overcome this potential error?
1

[DH = 467 kJ mol .


heat produced = mass specific heat capacity change in temperature.
Assume that the specific heat capacity of the solution is 4.18 J K

g ]

...........................................................................................................................
...........................................................................................................................
...........................................................................................................................
...........................................................................................................................
...........................................................................................................................
...........................................................................................................................
(4)

Sri Lankan School

39

(iv)

The most difficult thing to measure accurately is the time it takes for the
magnesium to disappear and the time measured can be up to 2 seconds out.
Assuming this error, calculate the shortest time at 56 C and the longest time at
10 C for this reaction.
Complete the table for these times. Plot the two points on the grid below and join
them with a straight line. From the gradient, which equals E A/R, of this line
calculate another value for the activation energy.

0 .0 0 3 0
1 .0

1/T
1
/K

Temperature
/ C

Temperature
/K

56

329

3.04 10

10

283

3.53 10

0 .0 0 3 1

0 .0 0 3 2

1 / T (K 1)
0 .0 0 3 3

0 .0 0 3 4

0 .0 0 3 5

time
/s

1/time
1
/s

ln 1/time

0 .0 0 3 6

1 .5

2 .0

ln 1 / t

2 .5

3 .0

3 .5

4 .0

4 .5

5 .0

Sri Lankan School

40

(v)

If the reaction mixture is not stirred, the magnesium tends to float on the surface of
the acid.
Suggest how this would affect the measurements of the rate of the reaction.
...........................................................................................................................
...........................................................................................................................
...........................................................................................................................
...........................................................................................................................
(1)

(vi)

Suggest two other improvements the student could do to this experiment to


improve the accuracy or validity of the results.
...........................................................................................................................
...........................................................................................................................
...........................................................................................................................
...........................................................................................................................
...........................................................................................................................
...........................................................................................................................
(2)

(vii) If ethanoic acid of the same concentration and at the same temperature is used
instead of hydrochloric acid, explain how the rate would differ.
...........................................................................................................................
...........................................................................................................................
...........................................................................................................................
(1)
(Total 24 marks)

Sri Lankan School

41

36.

One step in the production of nitric acid is the oxidation of ammonia.


4NH3 + 5O2 4NO + 6H2O
This is carried out at 900 C over a platinum-rhodium catalyst and is an example of
heterogeneous catalysis.
Explain in terms of collision frequency and collision energy how the rate would change if the
temperature were increased, and which of these causes the greater effect.
What is the difference between a heterogeneous and a homogeneous catalyst? Suggest one
advantage of using a heterogeneous catalyst in processes such as this.
...............................................................................................................................................
...............................................................................................................................................
...............................................................................................................................................
...............................................................................................................................................
...............................................................................................................................................
...............................................................................................................................................
...............................................................................................................................................
...............................................................................................................................................
...............................................................................................................................................
...............................................................................................................................................
...............................................................................................................................................
...............................................................................................................................................
...............................................................................................................................................
...............................................................................................................................................
...............................................................................................................................................
...............................................................................................................................................
(Total 6 marks)

Sri Lankan School

42

37.

In a standard hydrogen electrode


A

the hydrogen gas is at one atmosphere pressure

a solution of 1 mol dm sulfuric acid is used

a temperature of 273 K is maintained

a piece of shiny platinum foil is used

(Total 1 mark)

38.

For a redox reaction to be thermodynamically feasible, E cell must be


A

positive

negative

greater than +0.3 V

more negative than 0.3 V


(Total 1 mark)

39.

The standard electrode potential for the electrode system based on the equation below is
+1.51 V.

MnO4 (aq) + 8H (aq) + 5e

2+

Mn (aq) + 4H2O(l)

Which of the following statements about the electrode system is correct?


A

the electrode potential at pH 5 is +1.51 V.

Mn (aq) is acting as an oxidising agent.

changing the concentration of Mn (aq) would cause a change in the electrode potential.

the electrode used in this half-cell is made of manganese.

2+

2+

(Total 1 mark)

Sri Lankan School

43

40.

Which of the following is always proportional to Ecell for a chemical reaction?


A

DHr

DSsystem

DSsurroundings

DStotal
(Total 1 mark)

41.

Methanol can be vigorously oxidised with an acidified solution containing dichromate(VI) ions,
2
Cr2O7 , to form methanoic acid and chromic(III) ions.
(a)

What are the oxidation numbers of carbon in methanol and methanoic acid?
Methanol
Methanoic acid
A

+1

+2

+1

+2

2
(1)

(b)

How many moles of methanol react with one mole of dichromate(VI) ion, Cr 2O7 ?
A

3
(1)
(Total 2 marks)

Sri Lankan School

44

42.

(a)

(i)

Give the electron configuration of:


Fe
Fe

[Ar] .........................................................................................................
2+

[Ar] .........................................................................................................
(1)

(ii)

2+

Draw the structure of the hexaaquairon(II) ion, [Fe(H2O)6] , clearly showing its
shape.

(1)

(iii)

Give the equation for the complete reaction of hydroxide ions with a solution of
hexaaquairon(II) ions.

(1)

(iv)

State what you would see if the product mixture in (iii) is left to stand in air.
...........................................................................................................................
...........................................................................................................................
(1)

Sri Lankan School

45

(b)

Consider the equation for the half reaction


Fe
(i)

2+

+ 2e

Fe

E = 0.44 V

Define the term standard electrode potential with reference to this electrode.
...........................................................................................................................
...........................................................................................................................
...........................................................................................................................
...........................................................................................................................
(3)

(ii)

Explain why the value of E suggests that the iron will react with an aqueous
2+
solution of an acid to give Fe ions and hydrogen gas.
...........................................................................................................................
...........................................................................................................................
...........................................................................................................................
...........................................................................................................................
...........................................................................................................................
...........................................................................................................................
...........................................................................................................................
(2)

(iii)

State why E values cannot predict that a reaction will occur, only that it is
possible.
...........................................................................................................................
...........................................................................................................................
(1)
(Total 10 marks)

Sri Lankan School

46

43.

This question is about the manufacture of sulphuric acid, H2SO4.


(a)

The first stage in the manufacture of sulphuric acid is the combustion of sulphur.
The following equation shows the reaction taking place when the standard enthalpy of
combustion of sulphur is measured.
S(s) + O2(g) SO2(g)
Define the term standard enthalpy of combustion.
.....................................................................................................................................
.....................................................................................................................................
.....................................................................................................................................
.....................................................................................................................................
(3)

Sri Lankan School

47

(b)

In the second stage of the manufacture of sulphuric acid, sulphur dioxide is oxidised to
sulphur trioxide as shown in the following equation:
2SO2(g) + O2(g)
(i)

2SO3(g)

H = 196 kJ mol

Explain, in terms of collision theory, why the rate of a reaction is increased by


increasing the temperature and by the addition of a catalyst.
Temperature
...........................................................................................................................
...........................................................................................................................
...........................................................................................................................
...........................................................................................................................
...........................................................................................................................
...........................................................................................................................
Catalyst
...........................................................................................................................
...........................................................................................................................
...........................................................................................................................
...........................................................................................................................
...........................................................................................................................
...........................................................................................................................
(4)

(ii)

State and explain the effect, if any, of increasing the temperature on the equilibrium
yield of sulphur trioxide.
...........................................................................................................................
...........................................................................................................................
...........................................................................................................................
...........................................................................................................................
...........................................................................................................................
...........................................................................................................................
(2)

Sri Lankan School

48

Sri Lankan School

49

(iii)

State and explain the effect, if any, of an increased pressure on the equilibrium
yield of sulphur trioxide.
...........................................................................................................................
...........................................................................................................................
...........................................................................................................................
...........................................................................................................................
...........................................................................................................................
...........................................................................................................................
...........................................................................................................................
...........................................................................................................................
(2)

(c)

The following equation represents the overall reaction for the manufacture of sulphuric
acid from sulphur, oxygen and water.
2S(s) + H2O + 3O2 2H2SO4(l)
Use the data below to calculate the enthalpy change for this reaction.
Substance

Hf

/ kJ mol
H2O (l)

286

H2SO4 (l)

814

(2)
(Total 13 marks)

Sri Lankan School

50

44.

Consider the equilibrium


N2O4(g)
(a)

2NO2(g)

H = +58 kJ mol

Write the expression for the equilibrium constant, Kp, for the above reaction.
(1)

(b)

(i)

An equilibrium mixture contains a mole fraction of dinitrogen tetroxide,


N2O4 = 0.20, and nitrogen dioxide, NO2 = 0.80. The total pressure of this mixture
is 1.1 atm.
Calculate Kp at this temperature, stating its units.

(3)

(ii)

Calculate the total pressure required to reduce the mole fraction of N 2O4 to 0.10.

(3)

(c)

(i)

What is the effect on Kp, if any, of raising the temperature?


...........................................................................................................................
(1)

Sri Lankan School

51

(ii)

Use your answer to (c)(i) to explain the effect of increasing the temperature on the
position of equilibrium.
...........................................................................................................................
...........................................................................................................................
...........................................................................................................................
...........................................................................................................................
...........................................................................................................................
...........................................................................................................................
(2)
(Total 10 marks)

45.

(a)

The values of the ionic product of water, Kw, at two different temperatures are shown in
the table below.
Temperature /C

Kw
2

/ mol dm

(i)

25

1.00 10

14

50

5.48 10

14

Write an equation to represent the ionisation of water.


...........................................................................................................................
(1)

(ii)

Write the expression for Kw.


...........................................................................................................................
(1)

(iii)

Define the term pH.


...........................................................................................................................
...........................................................................................................................

Sri Lankan School

52

(1)

Sri Lankan School

53

(iv)

Calculate the pH of pure water at 50 C.

(2)

(v)

Explain why pure water at 50 C is neutral despite the fact that its pH is not 7.
...........................................................................................................................
...........................................................................................................................
(1)

(b)

The pH curve shown below was obtained when a 0.100 mol dm solution of sodium
3
3
hydroxide was added to 25.0 cm of a 0.100 mol dm solution of ethanoic acid.
14
12
10
pH
8
6
4
2
0

10

20

V o lu m e 0 .1 0 0 m o l d m

Sri Lankan School

30
3

N aO H / cm

40
3

54

(i)

What volume of sodium hydroxide solution is required to neutralise half of the


ethanoic acid in this reaction?
Volume added = ..............................................................cm

(1)

(ii)

Use the graph to determine the pH when the volume of sodium hydroxide you have
stated in part (i) has been added.
pH is .....................................................................................
(1)

(iii)

Write an expression for the acid dissociation constant, Ka, of ethanoic acid,
CH3COOH.

(1)

Sri Lankan School

55

(iv)

Use your answers to parts (ii) and (iii) to determine the value of Ka for ethanoic
acid at the temperature of the titration. Give your answer to two significant figures.

(2)

(c)

Phenolphthalein is a suitable indicator for a titration between ethanoic acid and sodium
hydroxide solutions whereas methyl orange is not a suitable indicator.
Explain why this is so.
.....................................................................................................................................
.....................................................................................................................................
.....................................................................................................................................
.....................................................................................................................................
.....................................................................................................................................
.....................................................................................................................................
.....................................................................................................................................
(2)
(Total 13 marks)

46.

(a)

Define the term standard electrode potential, making clear the meaning of standard.
.....................................................................................................................................
.....................................................................................................................................
.....................................................................................................................................
.....................................................................................................................................
.....................................................................................................................................
(2)

Sri Lankan School

56

Sri Lankan School

57

(b)

Explain why a reference electrode is needed whenever a standard electrode potential is


measured.
.....................................................................................................................................
.....................................................................................................................................
.....................................................................................................................................
.....................................................................................................................................
(1)

(c)

Hydrogen peroxide decomposes in a disproportionation reaction.


(i)

Explain the meaning of disproportionation.


...........................................................................................................................
...........................................................................................................................
...........................................................................................................................
...........................................................................................................................
...........................................................................................................................
(2)

(ii)

Use the following information to derive the equation for the disproportionation of
hydrogen peroxide.

Calculate E

cell

and explain whether the reaction is thermodynamically feasible.

H2O2

E = + 0.68 V

2H2O

E = + 1.77 V

O2 + 2H + 2e
H2O2 + 2H + 2e

...........................................................................................................................
...........................................................................................................................
...........................................................................................................................
...........................................................................................................................
...........................................................................................................................
...........................................................................................................................
(3)

Sri Lankan School

58

(iii)

Explain why your answer to (ii) does not necessarily show that hydrogen peroxide
will disproportionate under standard conditions.
...........................................................................................................................
...........................................................................................................................
...........................................................................................................................
(1)
(Total 9 marks)

47.

(a)

The conversion of butan-2-ol to 2-bromobutane can be performed as outlined below:


Butan-2-ol is heated with a mixture of 50 % aqueous sulphuric acid and sodium
bromide for 45 minutes.
The crude 2-bromobutane is distilled off.
The crude 2-bromobutane is shaken with pure water, which removes the sulphuric acid
and some of the butan-2-ol that contaminates the product.
The organic layer is separated and then shaken with concentrated hydrochloric acid to
remove residual butan-2-ol.
The organic layer is then shaken with dilute sodium carbonate solution.
Anhydrous calcium chloride is added to the organic layer and allowed to stand for
some hours.
The organic layer is then redistilled in a dry apparatus.

(i)

Explain, in terms of kinetic factors, why the mixture is heated for a significant
amount of time.
...........................................................................................................................
...........................................................................................................................
(1)

(ii)

Why is sulphuric acid necessary in the reaction mixture?


...........................................................................................................................
...........................................................................................................................
...........................................................................................................................

Sri Lankan School

59

(2)

Sri Lankan School

60

(iii)

Suggest why butan-2-ol, which is only partially miscible with water, is much more
soluble in concentrated hydrochloric acid.
...........................................................................................................................
...........................................................................................................................
...........................................................................................................................
...........................................................................................................................
(2)

(iv)

Why is the organic layer shaken with dilute sodium carbonate solution?
...........................................................................................................................
...........................................................................................................................
(1)

(v)

What is the purpose of the anhydrous calcium chloride?


...........................................................................................................................
(1)

(vi)

How would you heat the mixture safely? Explain your choice of method.
...........................................................................................................................
...........................................................................................................................
...........................................................................................................................
(2)

Sri Lankan School

61

(b)

Both 2-bromobutane and butan-2-ol are chiral molecules.


If one optical isomer of 2-bromobutane is used to make butan-2-ol by reaction with
aqueous hydroxide ions, the product mixture is not optically active.
The mechanism for the reaction is either SN1 or SN2; these are given below

SN1

C H 2C H
H 3C

C H 2C H

B r

H 3C

H
C H 2C H
H 3C

+ :B r

H
C H 2C H

+ :O H

H 3C

SN2

O H

H O :

C H 2C H
H 3C

C H 2C H

B r

H O

C
H

C H 2C H
H O

B r

H 3C

C
C H

Sri Lankan School

+ :B r

62

Explain which one of these mechanisms is consistent with the lack of optical activity in
the product mixture.
.....................................................................................................................................
.....................................................................................................................................
.....................................................................................................................................
.....................................................................................................................................
.....................................................................................................................................
.....................................................................................................................................
.....................................................................................................................................
(3)

(c)

The oxidation of butan-2-ol with hot potassium dichromate(VI) in acidic solution


produces butanone, CH3COCH2CH3.
(i)

What would you see as the reaction proceeds?


...........................................................................................................................
(1)

(ii)

The dichromate(VI) ion is reduced under these conditions to chromium(III) ions.


The half-equation for the oxidation of butan-2-ol to butanone is
+

CH3CH(OH)CH2CH3 CH3COCH2CH3 + 2H + 2e

Write the ionic half-equation for the reduction of dichromate(VI) ions, and hence
derive the overall equation for the oxidation of butan-2-ol.
...........................................................................................................................
...........................................................................................................................
...........................................................................................................................
...........................................................................................................................
...........................................................................................................................
(2)

Sri Lankan School

63

(iii)

The IR spectra of butan-2-ol and of the organic product from its oxidation with
dichromate(VI) ions are given below.

S p e c tru m o f b u ta n -2 -o l

T ra n s m itta n c e /%

100

50

0
4000

3000

2000

1500

W a v e n u m b e r/c m

1000

500

S p e c tru m o f th e o rg a n ic p ro d u c t fro m th e o x id a tio n o f b u ta n -2 -o l

T ra n s m itta n c e /%

100

50

0
4000

3000

2000

1500

W a v e n u m b e r/c m
Bond

Wavenumber/cm

1000

500

Bond

Wavenumber/cm

CH (alkanes)

28503000

CO (alcohols,
esters)

10001300

CH (alkenes)

30003100

OH (hydrogenbonded alcohols)

32303550

C=O (aldehydes,
ketones, carboxylic
acids)

16801750

Sri Lankan School

OH (hydrogenbonded carboxylic
acids)

25003300

64

What evidence is there from the spectra that the reaction in part (ii) has occurred?
...........................................................................................................................
...........................................................................................................................
...........................................................................................................................
...........................................................................................................................
...........................................................................................................................
...........................................................................................................................
(2)

(d)

When potassium dichromate(VI) is dissolved in water, the following equilibrium is set up


2

Cr2O7 (aq) + H2O(l)


(i)

2CrO4 (aq) + 2H (aq)

If a solution of barium ions is then added to this solution, solid barium chromate,
BaCrO4, is precipitated; it is sparingly soluble in water, so the equilibrium given
below also exists in the solution
2+

Ba (aq) + CrO4 (aq)

BaCrO4(s)

Explain what happens to the pH when the barium ions are added.
...........................................................................................................................
...........................................................................................................................
...........................................................................................................................
...........................................................................................................................
(2)

Sri Lankan School

65

(ii)

If a solution of lead(II) ions is added instead of barium ions, solid PbCrO 4 is


precipitated. This is almost completely insoluble in water so all chromate(VI) ions
are removed from solution
2+

Pb (aq) + CrO4 (aq) PbCrO4(s)


State how the pH of this solution differs from your answer in part (i).
...........................................................................................................................
...........................................................................................................................
(1)
(Total 20 marks)

48.

Phosphorus(V) chloride dissociates as follows:


PCl5(s)

PCl3(l) + Cl2(g)

H = + 123.8 kJ mol

(a)

(i)

Substance

Standard entropy, S
1 1
/ J mol K

PCl5(s)

+ 166.5

PCl3(l)

+ 217.1

Cl2(g)

+ 165.0

Explain why the entropy of solid phosphorus(V) chloride, PCl 5, is smaller than the
entropy of liquid phosphorus(III) chloride, PCl3?
...........................................................................................................................
...........................................................................................................................
...........................................................................................................................
(1)

(ii)

Calculate S

system

for the forward reaction. Include a sign in your answer.

(1)

Sri Lankan School

66

Sri Lankan School

67

(iii)

Is the sign of S

system

as you would expect? Fully justify your answer.

...........................................................................................................................
...........................................................................................................................
...........................................................................................................................
...........................................................................................................................
(2)

(b)

Calculate S
answer.

surroundings

for the forward reaction at 298 K. Include a sign and units in your

(2)

(c)

(i)

Use your answers to calculate S


sign in your answer.

total

for the forward reaction at 298 K. Include a

(1)

(ii)

Comment on the position of equilibrium at 298 K.


...........................................................................................................................
...........................................................................................................................
...........................................................................................................................
...........................................................................................................................
(1)

Sri Lankan School

68

Sri Lankan School

69

(d)

In an experiment to investigate this equilibrium, 41.7 g of phosphorus(V) chloride (molar


1
mass 208.5 g mol ) was heated in a closed vessel at 150 C until equilibrium was
established. The final pressure was found to be 4.32 atm and 0.15 moles of
phosphorus(V) chloride remained. At this temperature all of the reactants and products
are gaseous.
(i)

Give the expression for the equilibrium constant, Kp, and its units at this
temperature.

(2)

(ii)

Complete the table


Substance

Moles at start

PCl5(g)

Moles at equilibrium

Partial pressure at
equilibrium, peq
/atm

0.15

PCl3(g)

Cl2(g)

Total number of moles at equilibrium


(3)

(iii)

Calculate Kp.

(1)

Sri Lankan School

70

(iv)

How would you expect the value of Kp to change, if at all, if the following changes
were made? Justify each of your answers.
A Only 20.85 g of phosphorus(V) chloride had been used.
...........................................................................................................................
...........................................................................................................................
...........................................................................................................................
B The temperature had been increased to 250 C.
...........................................................................................................................
...........................................................................................................................
...........................................................................................................................
(2)
(Total 16 marks)

49.

Human blood plasma is a buffer solution. It partly owes its buffer properties to carbon dioxide,
produced by respiration, dissolving in the blood for transportation to the lungs.
Carbon dioxide dissolves in water establishing the equilibrium
CO2(aq) + H2O(l)
(a)

(i)

H (aq) + HCO3 (aq)

Write the expression for Ka for this equilibrium and give its units.

Units ........................
(2)

(ii)

What is the relationship between pKa and Ka?

(1)

Sri Lankan School

71

(b)

Explain what is meant by a buffer solution.


.....................................................................................................................................
.....................................................................................................................................
.....................................................................................................................................
.....................................................................................................................................
.....................................................................................................................................
.....................................................................................................................................
(2)

(c)

HCO3 (aq) can act as an acid or a base.


CO2(aq) + H2O(l)

H (aq) + HCO3 (aq)

In this equilibrium, decide whether HCO3 (aq) is acting as an acid or as a base. Give the
reason for your decision.
.....................................................................................................................................
.....................................................................................................................................
.....................................................................................................................................
.....................................................................................................................................
(1)

(d)

A sprinter had the pH and total carbonate concentration, [CO 2 + HCO3 ], of his blood
plasma measured immediately before and after a race. The following results were
obtained.

pH

Total carbonate concentration, [CO2 + HCO3 ]


3

/ mol dm

Before race

7.4

2.52 10

After race

7.3

1.98 10

The pH of a buffer solution is given by the equation


pH pK a log

[acid]
[base]

For this equilibrium pKa is 6.5.


Sri Lankan School

72

(i)

[acid]
[base]
Use this information, together with the data in the table, to calculate
before
the race.

(2)

(ii)

The concentration of HCO3 is 0.0224 mol dm . Calculate the concentration of


CO2 before the race.

(1)

(iii)

Use your results from (d)(i) and (ii) to complete the table below which will allow
you to compare the results before and after the race.
[acid]
[base]

Before race
After race

[HCO3 ]
3

/ mol dm

[CO2]
3

/ mol dm

0.0224
0.158

0.0171

0.00270

Two hypotheses have been proposed to explain why vigorous exercise results in an
increase in blood plasma acidity (from 7.4 to 7.3).

Sri Lankan School

73

Greater muscle activity during a race requires:


either
Hypothesis I

the combustion of larger quantities of glucose, resulting in an


increase in dissolved carbon dioxide and hence an increase in
acidity.

or
Hypothesis II

partial oxidation of glucose to lactic acid and hence an increase in


acidity.

State, giving your reasons, which hypothesis is favoured by the data and your
calculations.
...........................................................................................................................
...........................................................................................................................
...........................................................................................................................
...........................................................................................................................
...........................................................................................................................
...........................................................................................................................
...........................................................................................................................
...........................................................................................................................
...........................................................................................................................
...........................................................................................................................
(2)
(Total 11 marks)

50.

This question is about ammonia, NH3, which is produced as shown in the following equation.
N2(g) + 3H2(g)
(a)

2NH3(g)

Use oxidation numbers to explain why this is a redox reaction.


.....................................................................................................................................
.....................................................................................................................................
.....................................................................................................................................
(2)

Sri Lankan School

74

Sri Lankan School

75

(b)

(i)

Use the average (mean) bond enthalpy data to calculate a value for the enthalpy
change for this reaction. You are reminded to show all your working.
Bond

Average bond enthalpy


1
/ kJ mol

NN

944

HH

436

NH

388

(3)

(ii)

The actual standard enthalpy change for this reaction is 92 kJ mol . Explain why
the value you calculated in (b)(i) is not the same as this.
...........................................................................................................................
...........................................................................................................................
(1)

Sri Lankan School

76

(iii)

At room temperature, a mixture of nitrogen and hydrogen is thermodynamically


unstable with respect to ammonia, but is kinetically stable.
Use the data in (b)(i) and (ii) to help you explain why this mixture is
thermodynamically unstable
...........................................................................................................................
...........................................................................................................................
...........................................................................................................................
...........................................................................................................................
...........................................................................................................................
...........................................................................................................................
kinetically stable
...........................................................................................................................
...........................................................................................................................
...........................................................................................................................
...........................................................................................................................
...........................................................................................................................
...........................................................................................................................
(3)

Sri Lankan School

77

(c)

The manufacturer of ammonia would like to achieve a high rate of reaction and a high
equilibrium yield of product.
(i)

State and explain, in terms of collision theory, TWO ways to increase the rate of
the reaction. An increase in pressure does not alter the rate in this process.
...........................................................................................................................
...........................................................................................................................
...........................................................................................................................
...........................................................................................................................
...........................................................................................................................
...........................................................................................................................
...........................................................................................................................
...........................................................................................................................
...........................................................................................................................
...........................................................................................................................
...........................................................................................................................
...........................................................................................................................
...........................................................................................................................
...........................................................................................................................
(6)

Sri Lankan School

78

(ii)

State and explain TWO ways to increase the equilibrium yield of ammonia.
...........................................................................................................................
...........................................................................................................................
...........................................................................................................................
...........................................................................................................................
...........................................................................................................................
...........................................................................................................................
...........................................................................................................................
...........................................................................................................................
...........................................................................................................................
...........................................................................................................................
...........................................................................................................................
...........................................................................................................................
(4)
(Total 19 marks)

51.

(a)

The equilibrium between hydrogen iodide, hydrogen and iodine was investigated by
sealing hydrogen iodide in glass tubes and heating the tubes at 698 K until equilibrium
was reached.
2HI(g)

H2(g)+ I2(g)

H = + 9.4 kJ mol

The glass tubes were cooled rapidly and then opened in a solution of potassium iodide so
that the concentration of iodine at equilibrium could be determined by titration.
(i)

Suggest why the reaction mixture was cooled rapidly.


...........................................................................................................................
...........................................................................................................................
...........................................................................................................................
...........................................................................................................................
...........................................................................................................................
(2)

Sri Lankan School

79

Sri Lankan School

80

(ii)

The expression for the equilibrium constant, Kc, for the above reaction is
[ H 2 ( g )][I 2 (g )]
Kc =

[ HI(g)]2

One of the tubes was found to contain iodine at a concentration of


4
3
5.0 10 mol dm .
3

Calculate the equilibrium concentration of hydrogen iodide, in mol dm .


The equilibrium constant, Kc, for the above reaction is 0.019 at 698 K.

(3)

(b)

In a different experiment, 1.0 mol of hydrogen and 1.0 mol of iodine were allowed to
reach equilibrium at 698 K.
H2(g) + I2(g)

2HI(g)

At equilibrium, 80% of the hydrogen was converted to hydrogen iodide at a total pressure
of 1.1 atm.
(i)

Write an expression for the equilibrium constant, Kp, for the reaction as shown.

(1)

Sri Lankan School

81

(ii)

Calculate the value of Kp.

(4)

(iii)

Explain why, in this case, Kp has no units.


...........................................................................................................................
...........................................................................................................................
(1)
(Total 11 marks)

52.

This question is about propanoic acid, CH3CH2COOH.


(a)

Propanoic acid is a weak acid which dissociates as follows


CH3CH2COOH(aq) + H2O(l)
(i)

CH3CH2COO (aq) + H3O (aq)

In the above equation there are two conjugate acid-base pairs.


Identify them by completing the sentences below
Formula of one acid is ......................................................................................
The formula of its conjugate base is .................................................................
Formula of the other acid is ..............................................................................
The formula of its conjugate base is .................................................................
(2)

Sri Lankan School

82

(ii)

Propanoic acid is a weak acid. Explain what is meant by the term weak acid.
Weak
...........................................................................................................................
...........................................................................................................................
Acid
...........................................................................................................................
...........................................................................................................................
(2)

(b)

The acid dissociation constant, Ka, for propanoic acid is 1.30 10 mol dm at 298 K.
(i)

Write the expression for the acid dissociation constant, Ka, for propanoic acid.

(1)

Sri Lankan School

83

(ii)

A solution of propanoic acid has a pH of 3.44 at a temperature of 298 K.


3

Calculate the concentration, in mol dm , of the propanoic acid solution. Show


clearly two assumptions you have made.
Calculation:

Assumptions:
...........................................................................................................................
...........................................................................................................................
...........................................................................................................................
...........................................................................................................................
...........................................................................................................................
...........................................................................................................................
(5)

Sri Lankan School

84

(c)

A mixture of sodium propanoate and propanoic acid acts as a buffer solution.


(i)

What is meant by a buffer solution?


...........................................................................................................................
...........................................................................................................................
...........................................................................................................................
(2)

(ii)

Calculate the pH of a buffer solution made by mixing 100 cm of 0.0100 mol dm


3
3
propanoic acid solution with 300 cm of 0.00500 mol dm sodium propanoate
solution at 298 K.
5

[Ka for propanoic acid is 1.30 10 mol dm at 298 K]

(3)
(Total 15 marks)

Sri Lankan School

85

53.

(i)

Phosphorus trichloride reacts with oxygen to form phosphorus oxychloride in an


equilibrium reaction.
PCl3(g) + O2(g)

POCl3(g)

H = 153.6 kJ mol

Suggest how you would adjust the temperature and pressure to increase the yield of this
reaction. Justify your answer in each case.
Temperature ...............................................................................................................
.....................................................................................................................................
.....................................................................................................................................
.....................................................................................................................................
Pressure ......................................................................................................................
.....................................................................................................................................
.....................................................................................................................................
.....................................................................................................................................
(2)

(ii)

State the effect of the adjustments you propose in part (i) on the rate of the reaction.
Temperature ...............................................................................................................
.....................................................................................................................................
Pressure ......................................................................................................................
.....................................................................................................................................
(2)

Sri Lankan School

86

(iii)

On the axes below, sketch the energy profiles of the reaction in (c)(i) with and without a
catalyst. Label the profiles.

E n e rg y

R e a c tio n p a th
(2)
(Total 6 marks)

54.

The equation below shows the equilibrium existing between nitrogen, oxygen and nitrogen
monoxide.
N2(g) + O2(g)

2NO(g)

The equilibrium constant, Kp, at 298 K is 5.0 10

(a)

(i)

31

Write an expression for the equilibrium constant, Kp, in terms of the partial
pressures of the three gases.

(1)

Sri Lankan School

87

(ii)

Why does the value for Kp have no units?


...........................................................................................................................
...........................................................................................................................
(1)

(b)

An equilibrium mixture of these three gases was found to contain nitrogen, at a partial
pressure of 0.87 atm, and oxygen, at a partial pressure of 0.23 atm.
(i)

Calculate the partial pressure exerted by the nitrogen monoxide.

(2)

(ii)

Deduce the value of the total pressure of the equilibrium mixture of gases.

(1)

(iii)

Assuming that the total pressure on the mixture of gases is doubled, what, if any,
would be the effect on the
partial pressure of nitrogen monoxide
......................................................................................................................................
......................................................................................................................................
equilibrium constant, Kp?
......................................................................................................................................
......................................................................................................................................
(2)

Sri Lankan School

88

(c)

Inside a car engine, air (a mixture of nitrogen and oxygen) is drawn in and, under the high
temperatures operating, the value of Kp increases dramatically.
This increase is also accompanied by an increase in the value of Stotal. Typical values of
Kp and Stotal are shown in the table below.
1

Kp

Temperature / K

Stotal / J mol K
31

580

96

298

5.0 10

1500

1.0 10

Although the value of Ssystem is unlikely to alter very much, the value for Ssurroundings
will change significantly.

(i)

At a temperature of 1500 K, Stotal is negative.


Does this mean that the reaction between nitrogen and oxygen cannot occur at this
temperature? Explain your reasoning.
...........................................................................................................................
...........................................................................................................................
...........................................................................................................................
(1)

(ii)

Why is the value for Ssystem for this equilibrium approximately constant when the
temperature rises above 298 K?
...........................................................................................................................
...........................................................................................................................
(1)

(iii)

What is the sign of Ssurroundings for an endothermic reaction? Justify your answer.
...........................................................................................................................
...........................................................................................................................
(1)

Sri Lankan School

89

(iv)

Explain why an endothermic reaction results in an increase in the value of Stotal as


the temperature increases.
...........................................................................................................................
...........................................................................................................................
...........................................................................................................................
...........................................................................................................................
...........................................................................................................................
(1)

(d)

A student used the value for Kp at 1500 K to calculate the partial pressure of nitrogen
monoxide inside a working car engine.
Why might the actual partial pressure be lower than the calculated answer?
.....................................................................................................................................
.....................................................................................................................................
.....................................................................................................................................
(1)
(Total 12 marks)

Sri Lankan School

90

55.

Calculate the pH of the buffer solution formed by mixing 10 cm of aqueous benzoic acid of
3
3
concentration 0.010 mol dm with 40 cm of aqueous sodium benzoate of concentration 0.020
3
mol dm .
5

For benzoic acid, the acid dissociation constant, Ka, is 6.3 10 mol dm .
log

You may find it helpful to use the relationship pH = log Ka

[acid]
[base]

(Total 3 marks)

Sri Lankan School

91

56.

Hydrogen gas is manufactured by passing methane and steam over a heated catalyst.
CH4(g) + H2O(g)

CO(g) + 3H2(g)

H = +206 kJ mol

Explain the effect, if any, on the equilibrium yield of hydrogen by using:


(i)

a higher pressure
.....................................................................................................................................
.....................................................................................................................................
.....................................................................................................................................
.....................................................................................................................................
.....................................................................................................................................
(2)

(ii)

a higher temperature
.....................................................................................................................................
.....................................................................................................................................
.....................................................................................................................................
.....................................................................................................................................
.....................................................................................................................................
(2)

(iii)

a catalyst.
.....................................................................................................................................
(1)
(Total 5 marks)

Sri Lankan School

92

57.

(a)

Sketch the titration curve that you would expect if 25.0 cm of 0.100 mol dm
3
3
hydrochloric acid, HCl, is titrated with 40.0 cm of 0.100 mol dm ammonia solution,
3
NH .
14
12
10
pH
8
6
4
2
0
10

20
V o lu m e o f 0 .1 0 0 m o l d m
a m m o n ia s o lu tio n / c m 3

30

40

(4)

(b)

Using your answer to (a), select a suitable indicator for this titration. Put a tick in the
appropriate box in the table below.
Indicator

pKInd

thymol blue

1.7

bromocresol green

4.7

phenol red

7.9

phenolphthalein

9.3

( )

(1)

Sri Lankan School

93

(c)

Suggest why there is no suitable indicator for the titration of ethanoic acid with ammonia.
.....................................................................................................................................
.....................................................................................................................................
.....................................................................................................................................
.....................................................................................................................................
(2)
(Total 7 marks)

58.

Methanoic acid and ethanol react together to form ethyl methanoate, HCOOC 2H5, and water.
This reaction is reversible and can be allowed to reach equilibrium.
HCOOH(l) + C2H5OH(l)
(a)

HCOOC2H5 (l) + H2O(l)

H = +45 kJ mol

Draw the full structural formula of ethyl methanoate, showing all bonds.

(1)

(b)

What type of organic compound is ethyl methanoate?


.....................................................................................................................................
(1)

Sri Lankan School

94

(c)

In an experiment, 3.00 mol methanoic acid, HCOOH, and 6.25 mol ethanol, C2H5OH,
were mixed together. A small quantity of catalyst was added. The mixture was left for
several days in a water bath to reach equilibrium at constant temperature.
(i)

Complete the table.


Number of moles in the reaction mixture
HCOOH

C2H5OH

HCOOC2H5

H2O

at start of
experiment

3.00

6.25

0.00

0.00

at equilibrium

0.50
(2)

(ii)

Write an expression for the equilibrium constant, Kc, for the reaction.

(1)

(iii)

Calculate Kc for the reaction at the temperature of the experiment. The total volume
3

of the equilibrium mixture was 485 cm .

(2)

(iv)

State and explain whether Kc for this reaction has units.


...........................................................................................................................
...........................................................................................................................
(1)

Sri Lankan School

95

Sri Lankan School

96

(d)

(i)

The temperature of this equilibrium mixture is lowered.


Explain the effect of this on the value of the equilibrium constant and hence on the
yield of ethyl methanoate.
...........................................................................................................................
...........................................................................................................................
...........................................................................................................................
...........................................................................................................................
...........................................................................................................................
...........................................................................................................................
...........................................................................................................................
(4)

(ii)

A student added more catalyst to the mixture.


State, giving a reason, what would happen to the composition of the equilibrium
mixture.
...........................................................................................................................
...........................................................................................................................
(1)
(Total 13 marks)

59.

Brass is a widely-used alloy that contains copper and zinc. There are many varieties of brass
with different compositions.
In the volumetric analysis of the composition of brass, the first step is to react a weighed sample
of the alloy with nitric acid. This gives a greenish-blue solution.
(a)

The following standard electrode potentials are needed for this question:

2+

Zn + 2e
2+

Cu + 2e

NO3 + 2H + e

Sri Lankan School

Zn
Cu
NO2 + H2O

E /V
0.76
+ 0.34
+ 0.81

97

(i)

Use the half equations given above and the values of E to calculate the standard
electrode potential for the reaction between zinc and nitric acid and derive the
equation.
...........................................................................................................................
...........................................................................................................................
...........................................................................................................................
...........................................................................................................................
(2)

(ii)

Suggest why zinc does not produce hydrogen with nitric acid.
...........................................................................................................................
...........................................................................................................................
...........................................................................................................................
(2)

(iii)

If the greenish-blue solution is diluted with water it turns light blue and contains
hydrated copper(II) ions.
Name the light blue complex ion and draw its structure so as to show its shape.
Name .................................................................................................................
Structure:

(2)

Sri Lankan School

98

(iv)

If concentrated hydrochloric acid is added to a portion of the light blue solution it


turns green.
State the type of reaction that occurs and give an equation for the reaction.
...........................................................................................................................
...........................................................................................................................
(2)

(b)

The light blue solution from (a)(iii) is then neutralised, and reacted with an excess of
potassium iodide solution.
The following standard electrode potentials are needed:

2+

Cu + e

I2 + 2e

(i)

Cu

2I

E /V
+ 0.15
+ 0.54

Use these E values to explain why you would not expect the following reaction to
occur.
2+

2Cu (aq) + 4I (aq) 2CuI(s) + I2(aq)


...........................................................................................................................
...........................................................................................................................
...........................................................................................................................
(1)

(ii)

Explain why, in practice, the reaction in (i) does occur and iodine is liberated.
...........................................................................................................................
...........................................................................................................................
...........................................................................................................................
(2)

Sri Lankan School

99

(iii)

When the precipitate formed in the reaction in (i) is filtered off and then dissolved
in concentrated aqueous ammonia, a colourless solution is produced.
Suggest the formula of the cation in this solution.
...........................................................................................................................
(1)

(iv)

If the colourless solution from (iii) is left to stand in air for some time, it turns blue.
State why this is so, naming the reactant responsible for the change.
...........................................................................................................................
...........................................................................................................................
...........................................................................................................................
(2)

(c)

In a determination of the composition of a sample of brass, 1.50 g of the alloy was treated
3
to give 250 cm of a neutral solution of copper(II) nitrate and zinc nitrate.
3

Excess potassium iodide solution was added to 25.0 cm portions of this solution, and the
3
liberated iodine titrated with 0.100 mol dm sodium thiosulphate solution. The mean titre
3
was 16.55 cm .
2+

2Cu (aq) + 4I (aq) 2CuI(s) + I2(aq)


2

2S2O3 (aq) + I2(aq) 2I (aq) + S4O6 (aq)

(i)

State which indicator you would use for the titration and the colour change seen at
the end point.
...........................................................................................................................
...........................................................................................................................
(2)

(ii)

Explain why the indicator is not added until the reaction is nearly complete.
...........................................................................................................................
...........................................................................................................................
(1)

Sri Lankan School

100

Sri Lankan School

101

(iii)

Calculate the percentage of copper by mass in this brass.

(5)
(Total 22 marks)

60.

When solid lead(II) sulphate is added to a solution of sodium iodide, the following equilibrium
is established:

PbSO4(s) + 2I (aq)

PbI2(s) + SO4 (aq)

The equilibrium constant, Kc, for this reaction may be found by adding an excess of solid
lead(II) sulphate to a known volume of a standard solution of sodium iodide. The mixture is left
to reach equilibrium at a constant temperature, T.
Ice-cold water is added to freeze the position of equilibrium and the mixture is then titrated with
standard silver nitrate solution.
3

In a typical experiment, excess lead(II) sulphate was added to 50.0 cm of 0.100 mol dm
3
3
sodium iodide solution. The whole equilibrium mixture required 31.0 cm of 0.100 mol dm
silver nitrate solution to react with the aqueous iodide ions.
The expression for Kc for this reaction is
2

[SO 4 ]
2
Kc = [ I ]

Sri Lankan School

102

(a)

Why is it not necessary to know the mass of the lead(II) sulphate used in the experiment?
.....................................................................................................................................
.....................................................................................................................................
.....................................................................................................................................
(1)

(b)

Give the ionic equation for the reaction between silver nitrate solution and aqueous iodide
ions to produce a precipitate of silver iodide, AgI.
.....................................................................................................................................
(1)

(c)

From the data given above, calculate the equilibrium amounts of the iodide and of the
sulphate ions in solution. Hence calculate the equilibrium concentration of these ions, and
the value of Kc for the reaction at temperature T, including the units, if any.

(8)
(Total 10 marks)

Sri Lankan School

103

61.

This question concerns the reactions of some compounds of nitrogen.


(a)

The ammonium ion reacts with water and behaves as an acid.


+

NH4 (aq) + H2O(l)


(i)

NH3(aq) + H3O (aq)

Identify the TWO conjugate acid-base pairs in the spaces provided.


acid 1 ................................... base 1 ...................................
acid 2 ................................... base 2 ...................................
(1)

(ii)

Write the expression for the acid dissociation constant, Ka, of the ammonium ion.

(1)

(iii)

A solution of ammonium chloride has a pH of 5.00 at 25C.


10
3
Ka for the ammonium ion is 5.62 10 mol dm at 25C.
Calculate the concentration of this solution. State any assumptions you have made.

(4)

Sri Lankan School

104

(iv)

Use the following table and your answer from part (iii) to suggest a suitable
indicator for the titration of ammonia solution with hydrochloric acid. Justify your
answer.
Indicator

pKIn

thymol blue

1.7

methyl red

5.1

phenolphthalein

9.3

...........................................................................................................................
...........................................................................................................................
(2)

(b)

Hydrogen cyanide is a weak acid in aqueous solution.


Write an equation to show why aqueous solutions of cyanide ions are alkaline.
.....................................................................................................................................
(1)

(c)

Hydrogen cyanide reacts with propanal as follows:


CH3CH2CHO + HCN CH3CH2CH(OH)CN
Propanal is reacted with a solution of potassium cyanide, KCN, containing a little dilute
sulphuric acid.
(i)

What type of reaction is this?


...........................................................................................................................
(1)

Sri Lankan School

105

(ii)

Give the mechanism for the reaction.

(3)

(iii)

It is important that the pH is neither too acidic nor too alkaline if a good yield of
the product is to be obtained. Explain why this is so.
...........................................................................................................................
...........................................................................................................................
...........................................................................................................................
...........................................................................................................................
(2)

(d)

In an investigation of the kinetics of the nucleophilic substitution reaction between 1chloropropane and potassium cyanide in aqueous ethanolic solution, the reaction was
found to be first order with respect to 1-chloropropane and first order with respect to
cyanide ions.
(i)

Give the rate equation for the reaction.


...........................................................................................................................
(1)

Sri Lankan School

106

(ii)

Write a mechanism for the reaction that is consistent with this rate equation.

(3)
(Total 19 marks)

62.

The main reactions involved in the conversion of synthesis gas to methanol in both methods are
CO(g) + 2H2(g)
CO2(g) + 3H2(g)

CH3OH(g)
CH3OH(g) + H2O(g)

H = 91 kJ mol

H = 49 kJ mol

Compare the old and the new methods by considering their operating conditions.
Discuss ONE advantage of the old method and THREE advantages of the new method. Justify
your answers.

(i)

Advantage of old method.


.....................................................................................................................................
.....................................................................................................................................
.....................................................................................................................................
(1)

Sri Lankan School

107

(ii)

Advantages of new method.


Advantage 1 ................................................................................................................
.....................................................................................................................................
.....................................................................................................................................
Advantage 2 ................................................................................................................
.....................................................................................................................................
.....................................................................................................................................
Advantage 3 ................................................................................................................
.....................................................................................................................................
.....................................................................................................................................
(3)
(Total 4 marks)

63.

Ammonia can be oxidised to form nitrogen(II) oxide and water according to the equation
4NH3(g) + 5O2(g) 4NO(g) + 6H2O(g)

H = 905.6 kJ mol .

In industry, the reaction is carried out at 1123 K with a platinum/rhodium catalyst.


The standard entropy of one mole of each substance in the equation, measured at 298 K, is
shown in the table below.
Substance

Sri Lankan School

S / J mol K

NH3 (g)

+192.3

O2 (g)

+205.0

NO (g)

+210.7

H2O (g)

+188.7

108

(a)

(i)

Use the values given to calculate the standard entropy change of the system,

S system, for this reaction. Include the sign and units in your final answer.

(2)

(ii)

Is the sign for your value for S

system

what you expected? Justify your answer.

...........................................................................................................................
...........................................................................................................................
...........................................................................................................................
...........................................................................................................................
(1)

(iii)

Calculate the entropy change of the surroundings, Ssurroundings, at 1123 K for this
reaction. Include the sign and units in your final answer.

(2)

(iv)

Calculate the total entropy change, Stotal, for this reaction at 1123 K. Include the
sign and units in your final answer. You may assume that Ssystem is unchanged at
high temperatures.

(1)

Sri Lankan School

109

(v)

What does your answer to (iv) tell you about the extent of the reaction at 1123 K?
Justify your answer.
...........................................................................................................................
...........................................................................................................................
(1)

(vi)

An energy profile was proposed to illustrate the effect of the catalyst on this
reaction. The proposal has two errors. Draw a corrected version on the axes below.

P ro p o sa l
U n c a ta ly s e d
p a th w a y

E n e rg y

4 N H 3(g ) +
5 O 2 (g )

4 N O (g ) +
6 H 2O (g )

C a ta ly s e d
p a th w a y
P ro g re s s o f re a c tio n
C o r r e c te d v e r s io n

E n e rg y

4 N H 3(g ) +
5 O 2(g )

P ro g re s s o f re a c tio n
(2)

Sri Lankan School

110

(b)

The oxidation of nitrogen(II) oxide leads to the following equilibrium


2NO(g) + O2(g)

2NO2(g)

The number of moles of each gas in a reaction mixture at equilibrium, at a pressure of


1.5atm, was found to be

(i)

Substance

Number of moles
at equilibrium

NO (g)

0.025

O2 (g)

0.025

NO2 (g)

4.95

Write the expression for the equilibrium constant, Kp, for this reaction.

(1)

(ii)

Calculate the mole fraction of each gas and hence the value of the equilibrium
constant, Kp, for this mixture. Include units, if required, in your answer.

(4)

(iii)

What does your answer to (ii) tell you about the position of the equilibrium?
Justify your answer.
...........................................................................................................................
...........................................................................................................................
...........................................................................................................................
(1)

Sri Lankan School

111

Sri Lankan School

112

(iv)

If the total pressure of the reaction mixture was increased, describe what would
happen to the value of the equilibrium constant, Kp, and the partial pressure of
NO2(g). In each case justify your answer.
Equilibrium constant, Kp.
...........................................................................................................................
...........................................................................................................................
...........................................................................................................................
...........................................................................................................................
Partial pressure of NO2(g).
...........................................................................................................................
...........................................................................................................................
...........................................................................................................................
...........................................................................................................................
(2)
(Total 17 marks)

64.

(a)

The table below shows the acid dissociation constants, Ka, of three carboxylic acids.

(i)

Acid

Structural formula

Ka/ mol dm

Chloroethanoic

CH2ClCO2H

1.3 10

Dichloroethanoic

CHCl2CO2H

5.0 10

Trichloroethanoic

CCl3CO2H

2.3 10

Write an expression for the acid dissociation constant, Ka, of chloroethanoic acid.

(1)

Sri Lankan School

113

(ii)

Calculate the pH of a 0.0010 mol dm solution of chloroethanoic acid, making the


usual assumptions.

(3)

(iii)

Which acid would have the lowest pH at a concentration of 0.0010 mol dm ?


Use both the data and the structure of the acids to justify your answer. No further
calculation is required.
...........................................................................................................................
...........................................................................................................................
...........................................................................................................................
...........................................................................................................................
(2)

(b)

Chloroethanoic acid, CH2ClCO2H, reacts with methanol, CH3OH, in the presence of a


sulphuric acid catalyst.
(i)

Draw the displayed formula and give the name of the organic product formed.
Displayed Formula

Name .................................................................................................................
(3)

Sri Lankan School

114

(ii)

What name is given to the functional group formed in this organic product?
...........................................................................................................................
(1)

(iii)

What type of reagent is methanol in this reaction? Explain why it is able to behave
in this way and describe how it attacks the chloroethanoic acid.
You may find it helpful to draw a diagram.

...........................................................................................................................
...........................................................................................................................
...........................................................................................................................
(3)

(iv)

How would you convert the organic product of the reaction between
chloroethanoic acid and methanol back into the original compounds?
...........................................................................................................................
...........................................................................................................................
...........................................................................................................................
(2)
(Total 15 marks)

Sri Lankan School

115

65.

(a)

Define the term standard enthalpy of formation.


.....................................................................................................................................
.....................................................................................................................................
.....................................................................................................................................
.....................................................................................................................................
.....................................................................................................................................
(3)

(b)

In the Haber process, ammonia is manufactured from nitrogen and hydrogen as shown in
the equation.
N2(g) + 3H2(g)
(i)

2NH3(g)

Use the bond enthalpies below to calculate the standard enthalpy of formation of
ammonia.
Bond

Bond enthalpy / kJ mol


2

+945

+436

NN in N
HH in H

NH in NH

+391

(4)

Sri Lankan School

116

(ii)

Draw a labelled enthalpy level diagram for the formation of ammonia in the Haber
process.

E n th a lp y

(2)

(iii)

State the temperature used in the Haber process and explain in terms of the rate of
reaction and position of equilibrium, why this temperature is chosen.
Temperature .......................
...........................................................................................................................
...........................................................................................................................
...........................................................................................................................
...........................................................................................................................
...........................................................................................................................
...........................................................................................................................
...........................................................................................................................
...........................................................................................................................
...........................................................................................................................
...........................................................................................................................
(3)

Sri Lankan School

117

(iv)

Identify the catalyst used in the Haber process and state what effect, if any, it has
on the equilibrium yield of ammonia.
Catalyst ................................................................
Effect on yield ...................................................................................................
(2)

(v)

Explain why it is necessary to use a catalyst in this process.


...........................................................................................................................
...........................................................................................................................
...........................................................................................................................
...........................................................................................................................
(1)

(c)

The pressure used in the Haber process is 250 atmospheres.


(i)

State and explain an advantage of increasing the pressure to 1000 atmospheres.


...........................................................................................................................
...........................................................................................................................
...........................................................................................................................
...........................................................................................................................
...........................................................................................................................
(2)

(ii)

Suggest a disadvantage of using a pressure of 1000 atmospheres.


...........................................................................................................................
...........................................................................................................................
...........................................................................................................................
...........................................................................................................................
(1)
(Total 18 marks)

Sri Lankan School

118

Sri Lankan School

119

66.

(a)

Methanoic acid, HCOOH, is a weak acid. Explain what is meant by the terms weak and
acid.
.....................................................................................................................................
.....................................................................................................................................
.....................................................................................................................................
(2)

(b)

Write a balanced equation, including state symbols, for the reaction between aqueous
solutions of methanoic acid and sodium carbonate.
.....................................................................................................................................
(2)

(c)

The following equilibrium is set up when methanoic acid dissociates in water:


HCOOH(aq) + H2O(1)

(i)

HCOO (aq) + H3O (aq)

There are two conjugate acid-base pairs in the above equation.


Identify them by completing the sentences below:
Formula of one acid is

.......................................................... .

The formula of its conjugate base is

.......................................................... .
(1)

Formula of the other acid is

.......................................................... .

The formula of its conjugate base is

.......................................................... .
(1)

(ii)

Sri Lankan School

Write the expression for the acid dissociation constant, Ka, for methanoic acid.

120

(1)

Sri Lankan School

121

(iii)

Calculate the pH of a 0.100 mol dm solution of methanoic acid at 298 K.


4
3
[Ka for methanoic acid is 1.60 10 mol dm at 298 K]

(3)

(d)

A buffer solution is made up by mixing equal volumes of 0.100 mol dm methanoic acid
3
and 0.400 mol dm sodium methanoate.
(i)

Calculate the pH of the buffer solution obtained.

(3)

Sri Lankan School

122

(ii)

Explain why the pH of this mixture of methanoic acid and sodium methanoate
remains almost constant when a small quantity of an acidic solution, containing
hydrogen ions, or a small quantity of an alkaline solution, containing hydroxide
ions, is added.
In your explanation include equations to show the effect of adding hydrogen ions
and of adding hydroxide ions.
...........................................................................................................................
...........................................................................................................................
...........................................................................................................................
...........................................................................................................................
...........................................................................................................................
...........................................................................................................................
...........................................................................................................................
...........................................................................................................................
...........................................................................................................................
...........................................................................................................................
...........................................................................................................................
...........................................................................................................................
...........................................................................................................................
...........................................................................................................................
...........................................................................................................................
...........................................................................................................................
...........................................................................................................................
...........................................................................................................................
(3)
(Total 16 marks)

Sri Lankan School

123

67.

One stage in the manufacture of sulphuric acid is


2SO2(g) + O2(g)

2SO3(g)

2
p SO
3

The equilibrium constant Kp =

(a)

2
p SO
p O2
2

10.0 mol of SO2 and 5.00 mol of O2 were allowed to react. At equilibrium, 90.0% of the
SO2 was converted into SO3.
(i)

Calculate the number of moles of SO2, O2 and SO3 present in the equilibrium
mixture.

(2)

(ii)

Calculate the mole fractions of SO2, O2 and SO3 at equilibrium.

(1)

Sri Lankan School

124

(iii)

Assuming that the total pressure of the equilibrium mixture was 2.00 atm, calculate
the partial pressures of SO2, O2 and SO3 at equilibrium.

(1)

(iv)

Calculate the value of Kp.

(2)

(b)

The reaction between sulphur dioxide and oxygen is exothermic.


(i)

State the effect, if any, on Kp of increasing the temperature at constant pressure.


...........................................................................................................................
(1)

Sri Lankan School

125

2
p SO
3

(ii)

p 2 p O2
Use your answer to (i), and the expression Kp = SO 2
to explain the effect
on the position of equilibrium of increasing the temperature at constant pressure.
...........................................................................................................................
...........................................................................................................................
...........................................................................................................................
...........................................................................................................................
...........................................................................................................................
...........................................................................................................................
...........................................................................................................................
(2)

(c)

The reaction was repeated at a higher pressure whilst maintaining a constant temperature.
(i)

State the effect, if any, of an increase in the total pressure on the value of Kp.
...............................................................................
(1)

(ii)

State the effect, if any, of this increase in pressure on the amount of sulphur
trioxide in the equilibrium mixture.
...............................................................................
(1)

(d)

State the effect, if any, of a catalyst on:


(i)

Kp
...............................................................................
(1)

(ii)

the equilibrium position.


...............................................................................
(1)
(Total 13 marks)

Sri Lankan School

126

Sri Lankan School

127

68.

(a)

What is the formula of the ion found in all acidic solutions?


.....................................................................................................................................
(1)

(b)

Which of the following substances would form an acidic solution in water?


A CH4
D NH3

B NaOH
E HNO3

C HCO2H

.....................................................................................................................................
(2)
(Total 3 marks)

69.

10.0 cm of a solution of butanoic acid, CH3CH2CH2CO2H, of concentration 0.00660 mol dm ,


was titrated with a solution of aqueous ammonia using a pH probe.
The pH was recorded throughout, and the results were plotted as shown below.
11
10
9
8
pH
7
6

Y
X

5
4
3
2
1
0
0

Sri Lankan School

10

20
30
40
V o lu m e o f a m m o n ia a d d e d / c m

50

60

128

(a)

(i)

Using the pH of butanoic acid from the graph, calculate the initial hydrogen ion
concentration.

(2)

(ii)

Write the expression for the acid dissociation constant, Ka, for an aqueous solution
of butanoic acid.

(1)

(iii)

Calculate the value of Ka making the usual assumptions. Give your answer to two
significant figures.

(2)

(b)

(i)

Write an equation for the reaction between butanoic acid and ammonia. State
symbols are not required.

(1)

(ii)

Name the two compounds, apart from water, which are present in the mixture
between X and Y shown on the graph.
...........................................................................................................................
(2)

Sri Lankan School

129

(iii)

What type of mixture is present between X and Y? What evidence is there for your
answer by reference to the graph?
...........................................................................................................................
...........................................................................................................................
...........................................................................................................................
...........................................................................................................................
(2)

(iv)

Explain why it is not possible to carry out this titration using an indicator.
...........................................................................................................................
...........................................................................................................................
...........................................................................................................................
(1)

(v)

Use the graph to estimate the end-point of the titration. Hence calculate the
concentration of the ammonia solution.

(2)
(Total 13 marks)

70.

(a)

Define the term standard enthalpy of formation.


.....................................................................................................................................
.....................................................................................................................................
.....................................................................................................................................
.....................................................................................................................................
(3)

(b)

The dissociation of phosphorus pentachloride is a reversible reaction.

Sri Lankan School

130

PCl5(g)

Sri Lankan School

PCl3(g) + Cl2(g)

131

(i)

Use the values of enthalpy of formation given to calculate H for the forward
reaction.
Hf /
kJ mol

PCl5(g)

399

PCl3(g)

306

(1)

(ii)

Explain, with reasons, the effect that raising the temperature would have on the
composition of the equilibrium mixture.
...........................................................................................................................
...........................................................................................................................
...........................................................................................................................
(2)

(iii)

Other than by changing the temperature, suggest how the amount of PCl 5 present at
equilibrium could be increased. Give a reason for your answer.
...........................................................................................................................
...........................................................................................................................
...........................................................................................................................
(2)
(Total 8 marks)

71.

(a)

(i)

Sri Lankan School

Calculate the pH of 0.050 mol dm hydrochloric acid.

132

(1)

Sri Lankan School

133

(ii)

Calculate the concentration of hydroxide ions, in mol dm , in this solution.


14
2
6
At this temperature, Kw = 1.00 10 mol dm .

(1)

(b)

Phosphoric(V) acid, H3PO4, is a weak acid, forming the following equilibrium in water:
H3PO4(aq) + H2O(l)
(i)

H2PO4 (aq) + H3O (aq)

Write an expression for the acid dissociation constant, Ka, for phosphoric(V) acid.

(1)

(ii)

Sri Lankan School

Given that a 0.500 mol dm solution of phosphoric(V) acid has a pH of 1.20,


calculate the value of Ka, stating its units.

Assume that there is no further dissociation of the H 2PO4 ion.

134

(4)

Sri Lankan School

135

(c)

The H2PO4 ion formed when phosphoric(V) acid is added to water can dissociate further
2

into HPO4 .

H2PO4 (aq) +
..................

(i)

H2O(l)
..................

HPO4 (aq) +
..................

H3O (aq)
..................

In the spaces below the equation, identify the acid base conjugate pairs.
(2)

(ii)

Explain why very little dissociation of the H2PO4 ion occurs in solutions of
phosphoric(V) acid.
...........................................................................................................................
...........................................................................................................................
...........................................................................................................................
...........................................................................................................................
(1)

Sri Lankan School

136

(d)

The change in pH when 25.0 cm of 0.100 mol dm phosphoric(V) acid is titrated with
sodium hydroxide solution of the same concentration can be seen on the graph below.
7

pH

10

20

30

V o lu m e o f 0 .1 0 0 m o l d m 3 s o d iu m
h y d ro x id e s o lu tio n a d d e d / c m 3

Sri Lankan School

137

From the list below, select a suitable indicator for this titration. Justify your choice.
pKIn
bromocresol green

4.7

bromothymol blue

7.0

phenolphthalein

9.3

.....................................................................................................................................
.....................................................................................................................................
.....................................................................................................................................
.....................................................................................................................................
.....................................................................................................................................
(2)
(Total 12 marks)

72.

This question concerns redox chemistry.


Consider the following data

E /V
3+

Fe (aq) + e

Fe (aq)

2+

+0.77

Cl2(aq) + 2e

2Cl (aq)

+1.36

Mn (aq) + 4H2O(l)

2+

+1.51

MnO4 (aq) + 8H (aq) + 5e

(a)

(i)

Use the data to explain why dilute hydrochloric acid is not used to acidify
solutions of potassium manganate(VII).
...........................................................................................................................
...........................................................................................................................
...........................................................................................................................
...........................................................................................................................
(2)

Sri Lankan School

138

(ii)

Explain why titrations involving potassium manganate(VII) solution do not require


the addition of an indicator.
...........................................................................................................................
...........................................................................................................................
...........................................................................................................................
(1)

(b)

(i)

The ionic equation for the oxidation of iron(II) ions by manganate(VII) ions in
acidic solution is

2+

2+

3+

MnO4 (aq) + 5Fe (aq) + 8H (aq) Mn (aq) + 4H2O(l) + 5Fe (aq)


Explain, in terms of the half equations listed above, why the ratio of
manganate(VII) ions to iron(ii) ions is 1 : 5 in this reaction.

(1)

Sri Lankan School

139

(ii)

Patients suffering from iron deficiency are often prescribed tablets containing
hydrated iron(II) sulphate, FeSO4.7H2O.
Some tablets, of total mass 6.00 g, were dissolved in distilled water and made up to
3
3
200 cm in a volumetric flask. 25.0 cm portions of this solution were titrated
3
against a 0.0200 mol dm solution of acidified potassium manganate(VII). The
3
mean titre was 20.10 cm .
Calculate the percentage of hydrated iron(II) sulphate in the tablets.
1

[Molar mass FeSO4.7H2O = 278 g mol ]

(5)

(c)

An important application of redox reactions is in car batteries. The electrolyte is aqueous


sulphuric acid and the standard electrode potentials involved are shown below.

E /V
2+

Pb(s)

Pb (aq) + 2H2O(l)

Pb (aq) + 2e
PbO2(s) + 4H (aq) + 2e

(i)

2+

0.13
+1.46

Calculate the standard e.m.f. of the cell.


...........................................................................................................................
...........................................................................................................................

Sri Lankan School

140

(1)

Sri Lankan School

141

(ii)

A single cell in a car battery has an e.m.f. of 2.00 V. Suggest why this value is
different from the answer calculated in (i).
...........................................................................................................................
...........................................................................................................................
(1)
(Total 11 marks)

73.

Iron is produced from the ore haematite, which contains iron(III) oxide.
Some standard enthalpies of formation are given below.
Compound

(a)

(i)

Hf / kJ mol

Fe2O3 (s)

822

CO (g)

110

CO2 (g)

394

Write the equation for the complete reduction of iron(III) oxide by carbon
monoxide.
Calculate the enthalpy change for this reaction.

(3)

Sri Lankan School

142

Sri Lankan School

143

(ii)

Iron(III) oxide can also be reduced by carbon.


Fe2O3 + 3C 2Fe + 3CO

H = +492 kJ mol

Explain whether the reduction process in (i) or (ii) is more likely to occur.
...........................................................................................................................
...........................................................................................................................
...........................................................................................................................
(1)

(b)

On heating, the following exothermic reaction occurs


3Fe(s) + 4H2O(g)

Fe3O4(s) + 4H2(g)

At 600 C, a mixture of iron and steam is allowed to reach equilibrium. The equilibrium
partial pressures of hydrogen and steam are 1.6 atm and 1.2 atm respectively.
(i)

Write the expression for the equilibrium constant, Kp, for the reaction. Calculate its
value and state the units.

(2)

Sri Lankan School

144

(ii)

State the effect, if any, on the value of Kp when the temperature is increased.
Justify your answer.
...........................................................................................................................
...........................................................................................................................
...........................................................................................................................
...........................................................................................................................
(2)

(c)

When an iron nail is left in a test tube of water containing a little phenolphthalein, it starts

to react and region X becomes pink, due to the formation of OH ions.


X

Z
Y

Write the half-equations for the reactions taking place in regions X and Y.
Name the green precipitate that forms in region Z.
.....................................................................................................................................
.....................................................................................................................................
.....................................................................................................................................
.....................................................................................................................................
.....................................................................................................................................
.....................................................................................................................................
(3)

Sri Lankan School

145

(d)

Anhydrous iron(III) chloride is made by passing dry chlorine gas over heated iron. It is
formed as a dark red covalent gas with formula Fe 2Cl6 and has a similar structure and
reactions to aluminium chloride.
Draw a diagram to show the structure of the Fe2Cl6 molecule. Label the types of bonding
present.
State the shape around each iron atom.

(3)

(e)

Hydrated iron(III) chloride is ionic and soluble in water.


(i)

Describe a test for aqueous Fe

3+

ions.

...........................................................................................................................
...........................................................................................................................
...........................................................................................................................
(2)

(ii)

Explain why an aqueous solution of hydrated iron(III) chloride is acidic.


...........................................................................................................................
...........................................................................................................................
...........................................................................................................................
...........................................................................................................................
(3)
(Total 19 marks)

Sri Lankan School

146

Sri Lankan School

147

74.

The Hess cycle below can be used to estimate the enthalpy change of formation, DHf, of the
unstable gaseous compound with the formula HOCl(g).

+ 6 6 7 k J m o l 1

H O C l(g )

G a s e o u s a to m s o f th e e le m e n ts
H

+ 5 8 9 k J m o l 1

E le m e n ts in th e ir s ta n d a rd s ta te s

(a)

(i)

Insert formulae, with state symbols, into the appropriate boxes, to show the correct
quantities of each element.
(1)

(ii)

Use the cycle to calculate a value for the enthalpy change of formation, H f
[HOCl(g)].

(1)

Sri Lankan School

148

(iii)

Assuming that the HO bond energy is +464 kJ mol , calculate a value for the O
Cl bond energy.

(1)

Sri Lankan School

149

(b)

(i)

Draw a dot and cross diagram for the HOCl molecule showing outer electrons
only.

(2)

(ii)

Predict the HOCl bond angle. Justify your answer.


Angle ................................................................................................................
Justification ......................................................................................................
...........................................................................................................................
...........................................................................................................................
...........................................................................................................................
...........................................................................................................................
(2)

Sri Lankan School

150

(c)

HOCl(g) can be made from chlorine(I) oxide by the reversible reaction


Cl2O(g) + H2O(g)

2HOCl(g)

What effect, if any, would an increase in pressure have on the proportion of HOCl(g) at
equilibrium? Justify your answer.
.....................................................................................................................................
.....................................................................................................................................
.....................................................................................................................................
.....................................................................................................................................
(2)
(Total 9 marks)

75.

Dichloroethanoic acid reacts with pent-1-ene as shown by the following equation:


CHCl2CO2H(l) + C5H10(l)

(a)

CHCl2CO2C5H11(l)

Give the name of the product of this reaction and also the name for the new functional
group it contains.
.....................................................................................................................................
.....................................................................................................................................
(2)

Sri Lankan School

151

(b)

In an experiment to determine the equilibrium constant, 1.00 mol of dichloroethanoic acid


3
was mixed with 2.30 mol of pent-1-ene. The total volume remained at 300 cm
throughout. When equilibrium had been reached, it was found that 0.40 mol of
dichloroethanoic acid was left.
(i)

List the steps in the experiment you would carry out to determine the concentration
of dichloroethanoic acid present at equilibrium.
...........................................................................................................................
...........................................................................................................................
...........................................................................................................................
...........................................................................................................................
...........................................................................................................................
...........................................................................................................................
...........................................................................................................................
...........................................................................................................................
...........................................................................................................................
...........................................................................................................................
...........................................................................................................................
(4)

(ii)

Give the expression for the equilibrium constant, K c, for this reaction.

(1)

Sri Lankan School

152

(iii)

Complete the table for the number of moles and concentrations at equilibrium.
Substance

Number of
moles at start

Number of moles
at equilibrium

Concentration at
equilibrium
3
/mol dm

CHCl2COOH

1.00

0.40

1.33

C5H10

2.30

CHCl2COOC5H11

0
(3)

(iv)

Calculate the value of Kc, and give its units.

(3)
(Total 13 marks)

Sri Lankan School

153

76.

One stage in the manufacture of sulphuric acid is the exothermic reaction


2SO2(g) + O2(g)
(a)

2SO3(g)

In a closed container this mixture of gases would be in dynamic equilibrium.


State the meaning of the words dynamic and equilibrium in this context.
.....................................................................................................................................
.....................................................................................................................................
.....................................................................................................................................
.....................................................................................................................................
(2)

(b)

(i)

State the conditions of temperature and pressure used industrially for the
manufacture of SO3.
...........................................................................................................................
...........................................................................................................................
(2)

(ii)

Justify the choice of temperature for this reaction in terms of yield and rate.
...........................................................................................................................
...........................................................................................................................
...........................................................................................................................
...........................................................................................................................
...........................................................................................................................
...........................................................................................................................
(3)

Sri Lankan School

154

(iii)

The yield of products would be greater if a higher pressure were to be used for the
reaction.
Suggest a reason why a higher pressure than you have given in (i) is not used.
...........................................................................................................................
...........................................................................................................................
...........................................................................................................................
(1)

(c)

(i)

Calculate H for the forward reaction, given the enthalpies of formation below.
Hf / kJ mol
SO2(g)

297

SO3(g)

395

O2(g)

(2)

(ii)

State why the enthalpy of formation of oxygen, O2(g), is zero.


...........................................................................................................................
...........................................................................................................................
(1)

(d)

(i)

State the formula of the catalyst used in the industrial process.


...............................................................
(1)

Sri Lankan School

155

(ii)

Draw an enthalpy level diagram to show the reaction profiles of the uncatalysed
and catalysed reactions.

(3)

(iii)

Explain how the catalyst increases the reaction rate.


...........................................................................................................................
...........................................................................................................................
...........................................................................................................................
...........................................................................................................................
(2)

(e)

Suggest why the sulphur trioxide produced is passed into concentrated sulphuric acid
rather than water to form sulphuric acid at the end of the process.
.....................................................................................................................................
.....................................................................................................................................
.....................................................................................................................................
(1)
(Total 18 marks)

Sri Lankan School

156

77.

The weak acid methanoic acid, HCOOH, sets up the following equilibrium in water at 298 K:
HCOOH(aq) + H2O(l)

H3O (aq) + HCOO (aq)


4

The acid dissociation constant, Ka, for methanoic acid at 298 K is 1.78 10 mol dm .

(a)

A 0.200 mol dm solution of methanoic acid has a pH of 2.2 at 298 K.


3

20.0 cm of this solution is titrated with 0.100 mol dm sodium hydroxide solution until
excess alkali has been added.
On the grid below, sketch the titration curve you would expect for this reaction.

14
12
10
pH

8
6
4
2
0
10

20
30
40
V o lu m e o f s o d iu m h y d ro x id e s o lu tio n / c m

50

60

(4)

Sri Lankan School

157

(b)

Equal volumes of 0.500 mol dm methanoic acid and 0.250 mol dm sodium
methanoate solution are mixed to make a buffer solution.
(i)

Define the term buffer solution.


...........................................................................................................................
...........................................................................................................................
...........................................................................................................................
...........................................................................................................................
(2)

(ii)

Calculate the pH of this buffer solution.

(3)

Sri Lankan School

158

(iii)

Explain, with the aid of equations, how this mixture acts as a buffer solution.
...........................................................................................................................
...........................................................................................................................
...........................................................................................................................
...........................................................................................................................
...........................................................................................................................
...........................................................................................................................
...........................................................................................................................
...........................................................................................................................
...........................................................................................................................
...........................................................................................................................
...........................................................................................................................
...........................................................................................................................
(4)
(Total 13 marks)

78.

(a)

Define the term standard electrode potential.


.....................................................................................................................................
.....................................................................................................................................
.....................................................................................................................................
.....................................................................................................................................
(2)

Sri Lankan School

159

(b)

When a metal is placed in a solution of its ions, the electrical potential set up between the
metal and the solution cannot be measured without using a reference electrode. Explain
why this is so.
.....................................................................................................................................
.....................................................................................................................................
.....................................................................................................................................
(1)

(c)

The following data will be required in this part of the question.

E /V
2+

Zn (aq) + 2e

0.76

2+

Fe(s)

0.44

Sn (aq) + 2e

2+

Sn(s)

0.14

O2(g) + 2H2O(1) + 4e

4OH (aq)

+0.40

Fe (aq) + 2e

(i)

Zn(s)

Write an overall equation for the first stage in the rusting of iron.
...........................................................................................................................
...........................................................................................................................
(2)

(ii)

Calculate E for the reaction in (i) and show that it is feasible.


...........................................................................................................................
...........................................................................................................................
(2)

Sri Lankan School

160

(iii)

Use the E values above to explain why zinc is used in preference to tin for
preventing corrosion of steel car bodies.
...........................................................................................................................
...........................................................................................................................
...........................................................................................................................
...........................................................................................................................
...........................................................................................................................
(3)
(Total 10 marks)

79.

(a)

The elements from scandium to zinc belong to the d-block. Some, but not all, of these
elements are transition elements.
(i)

What is meant by the term transition element?


...........................................................................................................................
...........................................................................................................................
(1)

(ii)

Which of the elements, from scandium to zinc inclusive, are in the d-block but are
not transition elements?
...........................................................................................................................
(1)

(b)

(i)

Complete the electronic configurations of the Fe


Fe

2+

Mn

2+

and Mn

2+

ions below.

[Ar] ..............................................................................................................

2+

[Ar] .............................................................................................................
(1)

Sri Lankan School

161

(ii)

2+

Suggest why Fe ions are readily oxidised to Fe


3+
readily oxidised to Mn ions.

3+

ions, but Mn

2+

ions are not

...........................................................................................................................
...........................................................................................................................
...........................................................................................................................
(2)

(c)

Draw a diagram to show the three-dimensional structure of the [Fe(CN) 6] complex ion.

(2)

(d)

A solution of potassium manganate(VII), KMnO4, can be standardised by titration with


arsenic(III) oxide, As2O3. In this reaction, 5 mol of arsenic(III) oxide are oxidised to

arsenic(V) oxide, As2O5, by 4 mol of manganate(VII) ions, MnO4 .


Calculate the final oxidation number of the manganese.
.....................................................................................................................................
.....................................................................................................................................
.....................................................................................................................................
.....................................................................................................................................
.....................................................................................................................................
.....................................................................................................................................
.....................................................................................................................................
(4)

Sri Lankan School

162

Sri Lankan School

163

(e)

Ammonium vanadate(V), NH4VO3, reacts with dilute sulphuric acid to form a solution
+

containing yellow VO2 ions.


(i)

Write an ionic equation for the reaction of the anion in NH4VO3 with dilute
sulphuric acid.
...........................................................................................................................
(1)

(ii)

Is the reaction in (i) a redox reaction? Justify your answer.


...........................................................................................................................
...........................................................................................................................
(1)

(iii)

Addition of zinc to the solution containing VO2 ions causes the colour to change
from yellow to green then to blue, followed by green again and finally violet.
State the formulae of the ions responsible for each of these colours.
The first green colour

...............................................................................

The second green colour

...............................................................................

The violet colour

................................................................................
(3)
(Total 16 marks)

Sri Lankan School

164

80.

Domestic bleaches contain sodium chlorate(I), NaOCl.


(a)

Write the ionic equation to show the disproportionation of the chlorate(I) ion. Use
oxidation numbers to explain the meaning of the term disproportionation in this reaction.
.....................................................................................................................................
.....................................................................................................................................
.....................................................................................................................................
.....................................................................................................................................
.....................................................................................................................................
.....................................................................................................................................
.....................................................................................................................................
(3)

(b)

Domestic bleaches are dilute solutions of sodium chlorate(I). The amount of ClO ions in
a sample can be found by reacting it with excess acidified potassium iodide solution.

ClO + 2I + 2H I2 + Cl + H2O
The iodine produced is then titrated with standard sodium thiosulphate solution.
3

10.0 cm of a domestic bleach was pipetted into a 250 cm volumetric flask and made
up to the mark with distilled water.
3

A 25.0 cm portion of the solution was added to excess acidified potassium iodide
solution in a conical flask.
3

This mixture was titrated with 0.100 mol dm sodium thiosulphate solution, using
starch indicator added near the end point.
3

The mean titre was 12.50 cm .

(i)

Give the colour change you would see at the end point.
...........................................................................................................................
...........................................................................................................................
(1)

Sri Lankan School

165

(ii)

The equation for the reaction between iodine and thiosulphate ions is
2

2S2O3 + I2 S4O6 + 2I

Calculate the amount (moles) of chlorate(I) ions in 1.00 dm of the original bleach.

(5)

(iii)

Use the equation below to calculate the mass of chlorine available from 1.00 dm
of the original bleach. Give your answer to 3 significant figures.

ClO + Cl + 2H Cl2 + H2O

(1)

Sri Lankan School

166

(c)

Sodium thiosulphate can be used to remove the excess chlorine from bleached fabrics.
2

S2O3 + 4Cl2 + 5H2O 2SO4

+ 10H + 8Cl

By considering the change in oxidation number of sulphur, explain whether chlorine or


iodine is the stronger oxidising agent when reacted with thiosulphate ions.
.....................................................................................................................................
.....................................................................................................................................
.....................................................................................................................................
.....................................................................................................................................
.....................................................................................................................................
.....................................................................................................................................
.....................................................................................................................................
.....................................................................................................................................
(2)

(d)

Starch-iodide paper can be used to test for chlorine. It contains starch and potassium
iodide.
Explain the reactions taking place when a piece of damp starch-iodide paper is put in a
gas jar of chlorine. State what you would see.
.....................................................................................................................................
.....................................................................................................................................
.....................................................................................................................................
.....................................................................................................................................
.....................................................................................................................................
.....................................................................................................................................
.....................................................................................................................................
.....................................................................................................................................
(2)
(Total 14 marks)

Sri Lankan School

167

81.

(i)

Draw the displayed formula of propene, C3H6.

(1)

(ii)

Draw a dot and cross diagram for propene. You should show outer shell electrons only.

(1)
(Total 2 mark)

82.

One step in the manufacture of nitric acid is the reaction between nitrogen(II) oxide and oxygen
to form nitrogen(IV) oxide.
2NO(g) + O2(g)
(a)

(i)

2NO2(g)

H = 114 kJ mol

Use the equation to suggest the sign of Ssystem for the forward reaction. Justify
your answer.
...........................................................................................................................
...........................................................................................................................
...........................................................................................................................
...........................................................................................................................
(2)

Sri Lankan School

168

(ii)

What is the sign of Ssurroundings for the forward reaction? Justify your answer.
...........................................................................................................................
...........................................................................................................................
...........................................................................................................................
...........................................................................................................................
(2)

(b)

(i)

Write the expression for Kp for this reaction.


What are the units of Kp in this reaction?

Units ..............................................
(2)

Sri Lankan School

169

(ii)

Suggest how the temperature and pressure could be altered to make nitrogen(IV)
oxide more economically. Justify your suggestions by considering both yield and
rate.
Temperature
...........................................................................................................................
...........................................................................................................................
...........................................................................................................................
...........................................................................................................................
...........................................................................................................................
Pressure
...........................................................................................................................
...........................................................................................................................
...........................................................................................................................
...........................................................................................................................
...........................................................................................................................
(4)

(c)

(i)

What property would allow you to follow the progress of this reaction? Justify your
answer.
...........................................................................................................................
...........................................................................................................................
...........................................................................................................................
...........................................................................................................................
...........................................................................................................................
(2)

Sri Lankan School

170

(ii)

In a series of experiments, the following results were obtained.


Experiment

[NO(g)]
3
/mol dm

[O2(g)]
3

/mol dm

Initial rate
3 1
/mol dm s

1.0 10

1.0 10

8.0 10

2.0 10

1.0 10

3.2 10

2.0 10

2.0 10

6.4 10

What is the order of the reaction with respect to NO(g)? Justify your answer.
...........................................................................................................................
...........................................................................................................................
...........................................................................................................................
(2)

What is the order of the reaction with respect to O 2(g)?


...........................................................................................................................
(1)

(iii)

What is the rate equation for this reaction?

(1)

(iv)

What is the overall order for this reaction?


....................................................................................
(1)

Sri Lankan School

171

(v)

Calculate the rate constant, k, for this reaction. Include units with your answer.

(2)

(d)

Suggest why this reaction takes place quickly at room temperature and pressure.
.....................................................................................................................................
(1)
(Total 20 marks)

83.

(a)

When silver carbonate is heated, it decomposes into silver oxide and carbon dioxide.
Ag2CO3(s)

Ag2O(s) + CO2(g)

At 227 C, the value of the equilibrium constant, Kp, is 1.48 atm.


(i)

Write the expression for the equilibrium constant, Kp.

(1)

(ii)

Sri Lankan School

What is the pressure of carbon dioxide gas when silver carbonate is heated to a
temperature of 227 C in a closed vessel?

172

(1)

Sri Lankan School

173

(b)

When nitrosyl chloride, NOCl, is heated, it dissociates reversibly into nitric oxide, NO,
and chlorine, Cl2, according to the equation
2NOCl(g)
(i)

2NO(g) + Cl2(g)

H = +75.6 kJ mol

Write the expression for the equilibrium constant, Kp, for this reaction.

(1)

Sri Lankan School

174

(ii)

1.00 mol of nitrosyl chloride was placed in a sealed container and heated to 500 C.
Equilibrium was reached when 22.0% of the nitrosyl chloride had dissociated. The
pressure in the vessel was 5.00 atm.
Calculate the value of Kp at this temperature, stating its units.

(5)

(iii)

State the effect of an increase in temperature on the value of the equilibrium


constant, Kp. Justify your answer.
......................................................................................................................
......................................................................................................................
......................................................................................................................
(2)

Sri Lankan School

175

(iv)

Hence suggest in which direction the position of equilibrium moves when the
temperature is increased. Justify your answer.
......................................................................................................................
......................................................................................................................
......................................................................................................................
(2)
(Total 12 marks)

84.

(a)

The first step in the esterification of ethanoic acid, CH 3COOH, by ethanol in the presence
of a small quantity of concentrated sulphuric acid, is the reaction

+
CH3COOH + H2SO4 CH3COOH2 + HSO 4

In the space below the equation, identify the two acid base conjugate pairs.
(2)

(b)

Ethanoic acid, CH3COOH, is a weak acid and dissociates in water according to the
equation
CH3COOH(aq) + H2O(l)

H3O (aq) + CH3COO (aq)

Its acid dissociation constant, Ka, is


[H 3 O ][CH 3 COO ]
5
3
[CH 3 COOH]
Ka =
= 1.74 10 mol dm (at 25 C)

Sri Lankan School

176

(i)

The concentration of a solution of ethanoic acid can be determined by titrating a


3
25.0 cm sample in a conical flask against a standard solution of sodium hydroxide.
State whether the pH at the end point is less than 7, 7, or more than 7, and hence
name a suitable indicator for this titration.
pH at end point ..............................................................................................
Indicator ........................................................................................................
(2)

(ii)

Ethanoic acid is only about 1% ionised in dilute solutions. Its enthalpy of


1
neutralisation is 55 kJ mol , whereas the enthalpy of neutralisation of a strong
1
acid, such as hydrochloric acid, is 57 kJ mol .
Explain why there is so little difference between these two values.
......................................................................................................................
......................................................................................................................
......................................................................................................................
......................................................................................................................
......................................................................................................................
......................................................................................................................
(3)

Sri Lankan School

177

(iii)

Calculate the pH of a 0.140 mol dm solution of ethanoic acid, clearly showing


the TWO assumptions that you have made.
Calculation

Assumptions
......................................................................................................................
......................................................................................................................
......................................................................................................................
(4)

(iv)

To 50.0 cm of the solution in (III) , an equal volume of a 0.200 mol dm solution


of potassium ethanoate was added. Calculate the pH of the buffer solution
obtained.

(3)
(Total 14 marks)

Sri Lankan School

178

85.

(a)

(i)

Give the electronic configuration of:


Fe
Fe

[Ar].................................................................................................
2+

[Ar].................................................................................................
(2)

(ii)

2+

Draw the structure of the hexaaquairon(II) ion, [Fe(H2O)6] , so as to clearly show


its shape.

(1)

(iii)

Give the equation for the complete reaction of sodium hydroxide solution with a
solution of hexaaquairon(II) ions.
......................................................................................................................
(1)

(iv)

State what you would see if the product mixture in (iii) is left to stand in air.
......................................................................................................................
......................................................................................................................
(1)

(v)

Give the equation for a reaction in which iron metal is used as a catalyst.
......................................................................................................................
(1)

Sri Lankan School

179

(b)

Consider the half reaction


Fe
(i)

2+

+ 2e

Fe

E = 0.44 V

Define the term standard electrode potential with reference to this electrode.
......................................................................................................................
......................................................................................................................
......................................................................................................................
......................................................................................................................
(3)

(ii)

Explain, with the aid of an equation, why the value of E suggests that iron will
2+
react with an aqueous solution of an acid to give Fe ions and hydrogen gas.
......................................................................................................................
......................................................................................................................
......................................................................................................................
......................................................................................................................
......................................................................................................................
......................................................................................................................
......................................................................................................................
(3)

(iii)

State why E values cannot predict that a reaction will occur, only that it is
possible.
......................................................................................................................
......................................................................................................................
(1)

Sri Lankan School

180

(c)

Use the following standard electrode potentials to explain why iron(III) iodide does not
exist in aqueous solution.
Fe

3+

+ e

I2 + 2e

Fe
2I

2+

E = +0.77 V

E = +0.54 V

......................................................................................................................
......................................................................................................................
......................................................................................................................
......................................................................................................................
......................................................................................................................
......................................................................................................................
(2)
(Total 15 marks)

86.

But-2-enoic acid, CH3CH=CHCOOH, can be converted into 3-oxobutanoic acid,


CH3COCH2COOH in a three step synthesis.

(a)

Step 1

CH3CH=CHCOOH

CH3CHBrCH2COOH

Step 2

CH3CHBrCH2COOH

CH3CH(OH)CH2COOH

Step 3

CH3CH(OH)CH2COOH

CH3COCH2COOH

(i)

Identify the reagents needed for each step.


Step 1 ...........................................................................................................
......................................................................................................................
Step 2 ...........................................................................................................
......................................................................................................................
Step 3 ...........................................................................................................
......................................................................................................................
(4)

Sri Lankan School

181

(ii)

But-2-enoic acid exists as two stereoisomers.


Draw these stereoisomers and explain why one does not convert to the other at
room temperature.

......................................................................................................................
......................................................................................................................
......................................................................................................................
(2)

(b)

Give the structural formula of the organic products of the reaction of


(i)

but-2-enoic acid with an alkaline solution of potassium manganate(VII).

(1)

(ii)

3-oxobutanoic acid with hydrogen cyanide, HCN, in the presence of a trace of


hydroxide ions.

(1)

(c)

3-oxobutanoic acid is a weak acid. The value of its acid dissociation constant, Ka, is
4

2.63 10 mol dm .
(i)

Sri Lankan School

Give the structural formula of the conjugate base of 3-oxobutanoic acid.

182

(1)

Sri Lankan School

183

In parts (ii) and (iii) you may use HX as the formula for 3-oxobutanoic acid.
(ii)

Explain how a mixture of 3-oxobutanoic acid and its sodium salt can act as a buffer
solution when a small amount of alkali is added.
......................................................................................................................
......................................................................................................................
......................................................................................................................
......................................................................................................................
......................................................................................................................
......................................................................................................................
......................................................................................................................
......................................................................................................................
(4)

Sri Lankan School

184

(iii)

Calculate the mass of solid sodium 3-oxobutanoate that must be added to 100 cm
3
of a 0.500 mol dm solution of 3-oxobutanoic acid in order to make a buffer
solution of pH 3.80.

(5)
(Total 18 marks)

Sri Lankan School

185

87.

(a)

Carboxylic acids react reversibly with alcohols, in the presence of a catalyst, to form an
ester and water.
When 24.8 g of ethane-1,2-diol, HOCH2CH2OH, was mixed with 66.0 g of ethanoic acid,
CH3COOH, in the presence of a catalyst, equilibrium was reached after 80.0% of the
3

ethane-1,2-diol had reacted. The total volume at equilibrium was 90.0 cm .


HOCH2CH2OH + 2CH3COOH

CH3COOCH2CH2OOCCH3 + 2H2O

Write the expression for the equilibrium constant, Kc, and calculate its value. You should
make clear what units, if any, there are for Kc.

(7)

(b)

Ethane-1,2-diol can be made from ethene in a two-stage process. The overall reaction is:

1
C2H4 + H2O + 2 O2 HOCH2CH2OH
1054 g of ethane-1,2-diol was obtained from 560 g of ethene.
Calculate the percentage yield of the process.

(2)

Sri Lankan School

186

Sri Lankan School

187

(c)

Polyesters can be formed from compounds with two functional groups.


(i)

Give the structural formula of a reagent that would react with ethane-1,2-diol to
make a polyester. Draw the structure of the polymer made from this reagent and
ethane-1,2-diol.

(3)

(ii)

Explain whether it would be sensible for protective clothing, made from this
polymer, to be used in an environment where acid spills are likely.
......................................................................................................................
......................................................................................................................
(1)

(d)

Explain why the ester methyl methanoate, HCOOCH3, has a much lower boiling
temperature than its isomer ethanoic acid, CH 3COOH, and why ethanoic acid has a lower
boiling temperature than propanoic acid, C2H5COOH.
................................................................................................................................
................................................................................................................................
................................................................................................................................
................................................................................................................................
................................................................................................................................
................................................................................................................................
................................................................................................................................
................................................................................................................................
................................................................................................................................
................................................................................................................................
(5)
(Total 18 marks)

Sri Lankan School

188

Sri Lankan School

189

88.

(a)

(i)

Draw a labelled Hesss Law cycle for the dissolving of solid calcium hydroxide in
water, and use it and the data below to calculate the lattice energy of calcium
hydroxide.
H/ kJ mol
2+

Enthalpy of hydration of Ca (g)

1650

Enthalpy of hydration of OH (g)

460

Enthalpy of solution of Ca(OH)2(s)

16.2

(4)

(ii)

State and explain the trend in solubility in water of the Group 2 hydroxides.
......................................................................................................................
......................................................................................................................
......................................................................................................................
......................................................................................................................
......................................................................................................................
......................................................................................................................
......................................................................................................................
......................................................................................................................
......................................................................................................................
......................................................................................................................
(4)

Sri Lankan School

190

(b)

The dissolving of sparingly soluble calcium hydroxide in water can be shown by the
equilibrium reaction
Ca(OH)2(s) + (aq)

2+

Ca (aq) + 2OH (aq)

H = 16.2 kJ mol

State and explain the effect on the solubility of calcium hydroxide of


(i)

increasing the temperature


......................................................................................................................
......................................................................................................................
......................................................................................................................
......................................................................................................................
(3)

(ii)

adding sodium hydroxide solution.


......................................................................................................................
......................................................................................................................
......................................................................................................................
......................................................................................................................
(3)

(c)

When concentrated sulphuric acid is added to solid calcium chloride, one acidic gas is
given off.
When concentrated sulphuric acid is added to solid calcium bromide, three gases are
given off.
Identify the three gases given off in the reaction with calcium bromide. Explain why only
one gas is given off in the reaction with calcium chloride.
................................................................................................................................
................................................................................................................................
................................................................................................................................
................................................................................................................................
(4)
(Total 18 marks)

Sri Lankan School

191

Sri Lankan School

192

89.

In the first stage of an industrial process for purifying nickel, carbon monoxide is passed over
impure nickel at 323 K. Gaseous nickel tetracarbonyl, Ni(CO) 4, is formed.
Ni(s) + 4CO(g)
(a)

(i)

Calculate S

system

Ni(CO)4(g)

H = 191 kJ mol

for this reaction given the following standard entropy values.

S
1 1
/J mol K

Substance
Ni(s)

+29.9

CO(g)

+197.6

Ni(CO)4(g)

+313.4

Include a sign and units in your answer.

(2)

(ii)

Refer to the equation above and comment on the sign of your answer.
................................................................................................................................
................................................................................................................................
................................................................................................................................
(1)

(iii)

Calculate S

surroundings

at 323 K. Include a sign and units in your answer.

(2)

Sri Lankan School

193

(iv)

Deduce the direction of this reaction at 323 K. Justify your answer.


................................................................................................................................
................................................................................................................................
(1)

(b)

(i)

Write the expression for the equilibrium constant, Kp, for this reaction.

(1)

(ii)

100 moles of gaseous carbon monoxide is mixed with excess solid nickel at 323 K
in a vessel kept at 1.00 atmosphere pressure. At equilibrium, 1.00 mole of the
carbon monoxide has reacted.
Complete the table below and then calculate the value of Kp at this temperature.
Include the units of Kp in your answer.
Substance

Moles at start

Ni(CO)4

CO

100

Moles at
equilibrium

Partial pressure, peq


/atm

99.0

(4)
Sri Lankan School

194

Sri Lankan School

195

(iii)

As Kp has such a small value, suggest THREE ways in which this industrial
process could be improved to increase profitability. Justify each of your
suggestions.
................................................................................................................................
................................................................................................................................
................................................................................................................................
................................................................................................................................
................................................................................................................................
................................................................................................................................
................................................................................................................................
................................................................................................................................
................................................................................................................................
(3)

(c)

The second stage of this process is to recover the nickel from the nickel tetracarbonyl,
Ni(CO)4. By considering your calculations of the entropy changes, suggest how this
could be done. Justify your suggestion.
.......................................................................................................................................
.......................................................................................................................................
.......................................................................................................................................
.......................................................................................................................................
.......................................................................................................................................
(2)
(Total 16 marks)

Sri Lankan School

196

90.

A step in the Contact Process, for the manufacture of sulphuric acid, is the catalytic oxidation of
sulphur dioxide:
2SO2(g) + O2(g)

2SO3(g)

DH = 196 kJ mol

If sulphur dioxide and oxygen are heated to a temperature of 450 C and at a pressure of 2 atm
in the presence of a catalyst of vanadium(V) oxide, a dynamic equilibrium is reached in which
about 98 % of the sulphur dioxide is converted into sulphur trioxide.
(a)

(i)

Explain the meaning of the term dynamic equilibrium.


......................................................................................................................
......................................................................................................................
......................................................................................................................
(2)

(ii)

State the effect on the percentage of sulphur dioxide converted, if extra oxygen is
added to the system in equilibrium.
......................................................................................................................
(1)

(iii)

State the effect on the percentage of sulphur dioxide converted, if the catalyst is
removed from the system in equilibrium.
......................................................................................................................
(1)

(b)

Use your understanding of kinetics and equilibria to justify the temperature used to obtain
an economic yield in the manufacture of sulphuric acid.
...............................................................................................................................
...............................................................................................................................
...............................................................................................................................
...............................................................................................................................
(4)
(Total 8 marks)

Sri Lankan School

197

91.

Pentanoic acid, C4H9COOH, is a weak acid with an acid dissociation constant,


5

Ka = 1.5 10 mol dm .
(i)

What is meant by the term weak in a weak acid?


...............................................................................................................................
...............................................................................................................................
(1)

(ii)

Write the expression for the Ka of C4H9COOH.

(1)

(iii)

Calculate the pH of a 0.100 mol dm solution of C4H9COOH.

(3)

Sri Lankan School

198

(iv)

On the grid below sketch the change in pH during the addition of 50.0 cm of
3
3
3
0.100 mol dm sodium hydroxide solution to 25 cm of 0.100 mol dm pentanoic acid
solution.

14
12
10
pH
8
6
4
2

10

20

30

V o lu m e o f s o d iu m h y d ro x id e a d d e d / c m

40

50

(4)

(v)

Suggest, with reasoning, a suitable indicator for the titration in (iv).


Indicator

pKind

Bromophenol blue

4.0

Methyl red

5.1

Thymol blue

8.9

Alizarin yellow

12.5

Indicator ................................................................................................................
Reason ...................................................................................................................
...............................................................................................................................
(2)
(Total 11 marks)

Sri Lankan School

199

92.

Consider the esterification of the amino acid alanine, which has the formula
CH3CH(NH2)COOH.
CH3CH(NH2)COOH + C2H5OH
(a)

(i)

CH3CH(NH2)COOC2H5 + H2O

Write the expression for Kc.

(1)

(ii)

Using the average bond enthalpies given below, show that enthalpy change, DH,
for this reaction would be calculated as zero.
Average bond enthalpies / kJ mol

CO 358
OH 464

(2)

Sri Lankan School

200

(iii)

On the basis that DH is zero, state what the effect of increasing the temperature
will be on:
the value of Kc ............................................................................................................
......................................................................................................................
the rate of attainment of equilibrium ..........................................................................
......................................................................................................................
(2)

(b)

Alanine exists as a zwitterion.


(i)

Suggest the structure of this zwitterion.

(1)

(ii)

Suggest why alanine has a relatively high melting temperature.


......................................................................................................................
......................................................................................................................
(1)

(iii)

Using equations, show how alanine can react with both acid and alkali.

(2)

Sri Lankan School

201

Sri Lankan School

202

(c)

The alanine molecule is chiral.


(i)

What is meant by the term chiral?


......................................................................................................................
......................................................................................................................
(1)

(ii)

Draw the TWO optical isomers of alanine.

(2)

(iii)

How could the optical isomers of alanine be distinguished from each other?
......................................................................................................................
......................................................................................................................
(1)
(Total 13 marks)

Sri Lankan School

203

93.

The reaction between nitrogen and hydrogen can be used to produce ammonia.
2

N2(g) + 3H (g)

2NH3(g)

H = 92.2 kJ mol

Standard entropies are given below

= +191.6 J mol K

= +130.6 J mol K

= +192.3 J mol K

S [N2(g)]
S [H2(g)]
S [NH3(g)]

(a)

Calculate the entropy change of the system, S


units in your answer.

system,

for this reaction. Include a sign and

(2)

(b)

Calculate the entropy change of the surroundings, S


and units in your answer.

surroundings,

at 298 K. Include a sign

(2)

Sri Lankan School

204

(c)

(i)

Calculate the total entropy change, S


your answer.

total,

at 298 K. Include a sign and units in

(1)

(ii)

Is this reaction feasible at 298 K? Justify your answer.


...............................................................................................................................
...............................................................................................................................
...............................................................................................................................
(1)

(d)

In industry the reaction is carried out at about 700 K using an iron catalyst and high
pressures.
(i)

The yield of ammonia produced at equilibrium is less at 700 K than at 298 K, if the
pressure remains constant. In terms of entropy, explain why this happens.
...............................................................................................................................
...............................................................................................................................
...............................................................................................................................
(1)

(ii)

Higher pressures increase the yield of ammonia at equilibrium. Suggest a reason


why pressures greater than 300 atmospheres are not routinely used.
...............................................................................................................................
...............................................................................................................................
...............................................................................................................................
...............................................................................................................................
(1)

Sri Lankan School

205

Sri Lankan School

206

(iii)

Iron is a heterogeneous catalyst. Explain what is meant by heterogeneous.


...............................................................................................................................
...............................................................................................................................
(1)
(Total 9 marks)

94.

(a)

(i)

Use an equation to define the term pH.


......................................................................................................................
(1)

(ii)

Explain how some solutions can have a negative pH.


......................................................................................................................
......................................................................................................................
(1)

(b)

The concentration of propanoic acid can be found by titrating a sample with standard
sodium hydroxide solution.
(i)

Calculate the pH of 0.100 mol dm propanoic acid at 25 C; the value of the


5
3
dissociation constant for the acid, Ka, is 1.30 10 mol dm .

(3)

Sri Lankan School

207

(ii)

Sketch with reasonable accuracy the titration curve that you would expect if
3
3
3
25.0 cm of 0.100 mol dm propanoic acid were to be titrated with 0.100 mol dm
sodium hydroxide solution.

14
12
10
8
6
4
2
0

(iii)

10
20
30
40
3 s o d iu m h y d ro x id e s o lu tio n /c m
V o lu m e o f 0 .1 0 0 m o l d m

50
3

What is the significance of the pH of the mixture when 12.5 cm of sodium


hydroxide had been added to the propanoic acid?
......................................................................................................................
(1)
(Total 10 marks)

95.

This question concerns the equilibrium


2NO(g)
(a)

N2(g) + O2(g)

DH = 180 kJ mol

Define the term partial pressure.


...............................................................................................................................
...............................................................................................................................
(1)

Sri Lankan School

208

(b)

(i) Write the expression for Kp for the above reaction.

(1)

(ii)

At 1600 C and 1.5 atm pressure NO is 99 % dissociated at equilibrium. Calculate


the value of Kp under these conditions.

(4)

(c)

State and explain the effect on Kp and hence on the position of equilibrium of decreasing
the temperature at constant pressure.
...............................................................................................................................
...............................................................................................................................
...............................................................................................................................
...............................................................................................................................
...............................................................................................................................
(3)

Sri Lankan School

209

Sri Lankan School

210

(d)

The reaction
Ni(s) + 4CO(g)

Ni(CO)4(g)

is used to purify nickel.


(i)

Write the expression for Kp for this system.

(1)

(ii)

In order to achieve a high equilibrium yield of Ni(CO) 4 should a low or a high


partial pressure of carbon monoxide be used? Explain your answer in terms of Kp.
......................................................................................................................
......................................................................................................................
......................................................................................................................
(2)
(Total 12 marks)

96.

(a)

(i)

Define the term standard electrode potential, making clear the meaning of
standard in this context.
......................................................................................................................
......................................................................................................................
......................................................................................................................
(2)

Sri Lankan School

211

(ii)

Explain the need for a standard reference electrode in electrochemical


measurements.
......................................................................................................................
......................................................................................................................
......................................................................................................................
(1)

(b)

The following data will be required in this part of the question.

Mn

H2O2

+0.68

2H2O

+1.77

MnO4 + 8H + 5e
O2 + 2H + 2e
H2O2 + 2H + 2e
(i)

2+

E /V
+1.52

+ 4H2O

Derive the equation for the reaction between hydrogen peroxide and

manganate(VII) ions in acidic solution and use E values to show that the reaction
is feasible.
......................................................................................................................
......................................................................................................................
......................................................................................................................
(3)

(ii)

If a sample of hydrogen peroxide was titrated with potassium manganate(VII)


solution, what would you see during the reaction, and then at the endpoint?
......................................................................................................................
......................................................................................................................
......................................................................................................................
(3)

Sri Lankan School

212

(iii)

Derive the equation for the disproportionation of hydrogen peroxide and explain, in
terms of oxidation states, why it is a disproportionation reaction.
......................................................................................................................
......................................................................................................................
......................................................................................................................
(3)
(Total 12 marks)

97.

The industrial processes involved in the production of poly(chloroethene) are summarised in the
flow chart:
ethane ethene 1,2-dichloroethane chloroethene poly(chloroethene)
(a)

(i)

Ethane is converted to ethene by dehydrogenation.


Write a balanced equation, including state symbols, for this equilibrium reaction.

(1)

(ii)

Explain why conditions of high pressure are less favourable for ethene production.
............................................................................................................................
............................................................................................................................
............................................................................................................................
............................................................................................................................
(2)

(b)

Draw a labelled diagram of an ethene molecule, showing the electron density distribution
in the s and p bonds between the carbon atoms.

(2)
Sri Lankan School

213

Sri Lankan School

214

(c)

Give a chemical test which would distinguish between ethane and ethene.
State the result of your test with ethene.
Test ..............................................................................................................................
Result ...........................................................................................................................
(2)

(d)

1,2-dichloroethane is formed from ethene by reaction with chlorine.


State the type and mechanism of this reaction.
Type ...................................................................................................................................
Mechanism .........................................................................................................................
(2)
(Total 9 marks)

98.

The equation below shows a possible reaction for producing methanol.

CO(g) + 2H2(g) CH3OH(l)

(a)

H = -129 kJ mol

The entropy of one mole of each substance in the equation, measured at 298 K, is shown
below.

(i)

Substance

S
-1 -1
/J mol K

CO(g)

197.6

H2(g)

130.6

CH3OH(l)

239.7

Suggest why methanol has the highest entropy value of the three substances.
.............................................................................................................................
.............................................................................................................................
(1)

Sri Lankan School

215

(ii)

Calculate the entropy change of the system, S

system,

for this reaction.

(2)

(iii)

Is the sign of S

system

as expected? Give a reason for your answer.

.............................................................................................................................
.............................................................................................................................
.............................................................................................................................
(1)

(iv)

Calculate the entropy change of the surroundings S

surroundings,

at 298 K.

(2)

(v)

Show, by calculation, whether it is possible for this reaction to occur spontaneously


at 298 K.

(2)

Sri Lankan School

216

(b)

When methanol is produced in industry, this reaction is carried out at 400 C and 200
atmospheres pressure, in the presence of a catalyst of chromium oxide mixed with zinc
oxide. Under these conditions methanol vapour forms and the reaction reaches
equilibrium. Assume that the reaction is still exothermic under these conditions.
CO(g) + 2H2(g)
(i)

CH3OH(g)

Suggest reasons for the choice of temperature and pressure.


Temperature ........................................................................................................
.............................................................................................................................
.............................................................................................................................
.............................................................................................................................
Pressure ...............................................................................................................
.............................................................................................................................
.............................................................................................................................
(3)

(ii)

The catalyst used in this reaction is heterogeneous. Explain this term.


.............................................................................................................................
.............................................................................................................................
(1)

(iii)

Write an expression for the equilibrium constant in terms of pressure, Kp, for this
reaction.
CO(g) + 2H2(g)

CH3OH(g)

(1)

Sri Lankan School

217

(iv)

In the equilibrium mixture at 200 atmospheres pressure, the partial pressure of


carbon monoxide is 55 atmospheres and the partial pressure of hydrogen is 20
atmospheres.
Calculate the partial pressure of methanol in the mixture and hence the value of the
equilibrium constant, Kp. Include a unit in your answer.

(2)

(c)

The diagram below shows the distribution of energy in a sample of gas molecules in a
reaction when no catalyst is present. The activation energy for the reaction is EA.

(i)

What does the shaded area on the graph represent?


.............................................................................................................................
(1)

(ii)

Draw a line on the graph, labelled EC, to show the activation energy of the
catalysed reaction.
(1)
(Total 17 marks)

Sri Lankan School

218

99.

(a)

Ammonia reacts with water as below:


+

NH3(aq) + H2O(l)

NH4 (aq) + OH (aq)

A 0.100 mol dm solution of ammonia has a pH of 11.13.


(i)

Identify the BronstedLowry acid/base conjugate pairs in the equation. Clearly


label which are acids and which are bases.
......................................................................................................................
......................................................................................................................
(2)

(ii)

Draw, on the axes below, a graph to show how the pH of the solution varies as
3
3
40 cm of 0.100 mol dm hydrochloric acid (a strong acid) is added slowly to
3
20 cm of the ammonia solution.

14
12
10
pH

8
6
4
2
0

10

20

30

V o lu m e o f H C l s o lu tio n / c m

40

50

(4)

Sri Lankan School

219

(iii)

Select, from the following list, the indicator which would be the most suitable for
this titration. Give a reason for your choice.
Indicator

pKind

Range

methyl red

5.1

4.26.3

bromothymol blue

7.0

6.07.6

phenolphthalein

9.3

8.210.0

Indicator: ......................................................................................................
Reason: .........................................................................................................
......................................................................................................................
(2)

(b)

Nitrous acid, HNO2, is a weak acid with an acid dissociation constant


4

Ka = 4.70 10 mol dm at 4 C.
HNO2(aq) + H2O(l)
(i)

H3O (aq) + NO2 (aq)

Write the expression for Ka.

(1)

Sri Lankan School

220

(ii)

Calculate the pH of a 0.120 mol dm solution of nitrous acid.

(3)

(iii)

Calculate the pH of a buffer solution made by adding 1.38 g of sodium nitrite,


3
3
NaNO2, to 100 cm of the 0.120 mol dm solution of nitrous acid
4

(Ka = 4.70 10 mol dm ).

(4)

Sri Lankan School

221

(iv)

Suggest why a mixture of nitrous acid and sodium nitrite can act as a buffer
solution whereas a solution of sodium nitrite on its own does not.
......................................................................................................................
......................................................................................................................
......................................................................................................................
......................................................................................................................
(2)
(Total 18 marks)

100. The concentration of iron(II) ions in a solution can be found by titration with standard
potassium manganate(VII) solution. In the reaction iron(II) ions are oxidised to iron(III)
ions.
If a solution contains both iron(II) and iron(III) ions, the concentration of each ion can be
found by:
titrating samples of the original solution with standard potassium manganate(VII) solution
reacting samples of the original solution with zinc and dilute sulphuric acid and then titrating
with the same potassium manganate(VII) solution.
The following standard electrode potentials are required:

E /V
2+

+ 2e

Fe

2+

+ 2e

Fe

3+

+e

Zn

Zn

0.76

Fe

0.44

Fe
+

2+

MnO4 + 8H + 5e

Sri Lankan School

+ 0.77
Mn

2+

+ 4H2O

+ 1.51

222

(a)

(i)

Use suitable E values to show that both iron(II) and iron(III) ions in solution
should react with zinc to give iron metal.
......................................................................................................................
......................................................................................................................
......................................................................................................................
......................................................................................................................
(3)

(ii)

In practice the reaction produces only iron(II) ions and no iron metal.
2Fe

3+

+ Zn 2Fe

2+

+ Zn

2+

Suggest a reason for this.


......................................................................................................................
......................................................................................................................
(1)

(b)

(i)

Derive the ionic equation for the reaction between iron(II) ions and

manganate(VII), MnO4 , ions.


......................................................................................................................
......................................................................................................................
......................................................................................................................
(2)

(ii)

State what you would see as iron(II) ions in solution are titrated with potassium
manganate(VII). How would you detect the endpoint of the titration?
......................................................................................................................
......................................................................................................................
......................................................................................................................
......................................................................................................................
(3)

Sri Lankan School

223

(c)

A solution containing both iron(II) and iron(III) ions was titrated with 0.0200 mol dm
3
potassium manganate(VII) solution, 18.20 cm being required.

Another portion of the same volume of the same solution was reacted with zinc, and then
3
titrated with the same potassium manganate(VII) solution; 25.30 cm was required.
What mass of zinc had reacted?

(5)

(d)

(i)

Explain, including an equation, why aqueous solutions of hexaaqua ions such as


2+
[Fe(H2O)6] are acidic.
......................................................................................................................
......................................................................................................................
......................................................................................................................
......................................................................................................................
(3)

(ii)

Suggest with reasons which of 0.1 mol dm aqueous solutions of [Fe(H2O)6]


[Fe(H2O)6]

3+

2+

and

would be the more acidic.

......................................................................................................................
......................................................................................................................
......................................................................................................................
......................................................................................................................
(3)
(Total 20 marks)

Sri Lankan School

224

Sri Lankan School

225

101. When dinitrogen tetroxide, N2O4, dissociates, the following equilibrium is established.
N2O4(g)
(a)

2NO2(g)

State a property which could be measured to follow the progress of this reversible
reaction.
....................................................................................................................................
(1)

(b)

Write an expression for the equilibrium constant, Kc, for this reaction.

(1)

(c)

When a sample of 0.0370 moles of gaseous dinitrogen tetroxide is allowed to dissociate


3
at 25 C in a container of volume 1 dm , 0.0310 moles of N2O4(g) remain in the
equilibrium mixture.
Complete the table below, and use the data to calculate Kc for the reaction. Include a unit
in your answer.

Number of moles at start


3

Number of moles in 1 dm
at equilibrium

N2O4

NO2

0.0370

0.0310

Kc calculation:

(3)

Sri Lankan School

226

(d)

The reaction was repeated at a higher pressure, maintaining the temperature at 25 C.


(i)

How does this increase in pressure affect the amount of nitrogen dioxide, NO 2(g),
in the equilibrium mixture?
..........................................................................................................................
(1)

(ii)

How does this increase in pressure affect the value of Kc?


..........................................................................................................................
(1)

(e)

The reaction was repeated at the original pressure, but the temperature was increased to
75 C. The value of Kc was approximately twenty times greater.
How does this information show that the reaction is endothermic?
....................................................................................................................................
....................................................................................................................................
....................................................................................................................................
(1)

(f)

Predict the sign of Ssystem for the reaction, giving a reason for your answer.
....................................................................................................................................
....................................................................................................................................
....................................................................................................................................
(2)

(g)

Write the equation for the relationship between Ssurroundings and H for the reaction.

(1)

Sri Lankan School

227

Sri Lankan School

228

(h)

The magnitude of Ssystem for the reaction is greater than the magnitude of Ssurroundings.
Explain why this must be the case.
....................................................................................................................................
....................................................................................................................................
....................................................................................................................................
....................................................................................................................................
(2)
(Total 13 marks)

102. Crude oil is a mixture of compounds including members of the homologous series of alkanes. It
is used to manufacture fuels, such as petrol, and petrochemicals, such as buta-1,3-diene.
(a)

Define the term homologous series.


...............................................................................................................................
...............................................................................................................................
...............................................................................................................................
...............................................................................................................................
(3)

(b)

One of the reactions in the catalytic converter in the exhaust system of a car engine is
2NO(g) + 2CO(g)

N2(g) + 2CO2(g)

DH = 745 kJ mol

This reaction converts two poisonous gases into two harmless gases. The temperature in
the catalytic converter is high.

(i)

State, with a reason, which way the position of the equilibrium would shift if the
temperature were lowered.
......................................................................................................................
......................................................................................................................
......................................................................................................................
(2)

Sri Lankan School

229

(ii)

The gases from the engine are not cooled before entering the converter.
Explain why this is so.
......................................................................................................................
......................................................................................................................
(2)

(c)

Buta-1,3-diene is used in the manufacture of rubber.


(i)

Write the full structural formula of buta-1,3-diene.

(1)

(ii)

Explain whether or not buta-1,3-diene exists as geometric isomers.


......................................................................................................................
......................................................................................................................
(1)

Sri Lankan School

230

(iii)

Predict the structural formula of the organic product of the reaction of buta-1,3diene with an excess of an alkaline solution of potassium manganate(VII).

(2)
(Total 11 marks)

103. Propanoic acid is a weak acid which dissociates according to


CH3CH2COOH(aq) + H2O(l)

(a)

(i)

CH3CH2COO (aq) + H3O (aq)

Indicate, in the space provided below the equation, the two acid/base conjugate
pairs.
(2)

(ii)

Write the expression for the acid dissociation constant, Ka, for propanoic acid.

(1)
Sri Lankan School

231

Sri Lankan School

232

(iii)

Calculate the pH of a 0.100 mol dm solution of propanoic acid, for which


5
3
Ka = 1.3 10 mol dm .

(3)

(iv)

Calculate the concentration of hydroxide ions, OH , in this same solution of


14
2
6
propanoic acid. Kw = 1.00 10 mol dm at the temperature of the solution.

(3)

(b)

If sodium propanoate is dissolved in water, the pH of the resulting solution is not 7, but is
near to 8. By writing the equation for the reaction occurring suggest why this is so.
................................................................................................................................
................................................................................................................................
................................................................................................................................
(2)

Sri Lankan School

233

(c)

A mixture of sodium propanoate and propanoic acid behaves as a buffer solution.


(i)

What is meant by a buffer solution?


......................................................................................................................
......................................................................................................................
......................................................................................................................
(2)

(ii)

Calculate the pH of a buffer solution that is made by mixing equal volumes of


3
3
0.0500 mol dm propanoic acid and 0.100 mol dm sodium propanoate.

(3)
(Total 16 marks)

104. (a)

(i)

A fluoride of phosphorus, Y, contains 24.6% by mass of phosphorus and has a


1
molar mass of 126 g mol . Deduce the molecular formula of Y.
(4)

(ii)

Draw the shape of a molecule of compound Y. Show the values of the bond angles
on the diagram.
(3)

(iii)

Compound Y was converted into the ion PF6 . Draw and name the shape of PF6
and suggest a value for the bond angles.

(3)

Sri Lankan School

234

(b)

(i)

Consider the following data, which shows the formulae and boiling temperatures of
the Group 7 hydrides.
Group 7 hydride

Boiling temperature/C

HF

19

HCl

85

HBr

68

HI

35

Suggest why hydrogen fluoride, HF, has the highest boiling temperature of the
Group 7 hydrides.
(3)

(c)

When hydrogen fluoride is dissolved in water a solution of a weak acid is formed. Write
the expression for the acid dissociation constant, Ka, for hydrogen fluoride. Calculate the
3

value of Ka, with units, given that a 0.150 mol dm solution of hydrogen fluoride has a
pH of 2.04.
(5)
(Total 18 marks)

105. (a)

Consider the following equilibrium, which illustrates one industrial method used to
produce hydrogen:
CH4(g) + 2H2O(g)

CO2(g) + 4H2(g)

In a certain experiment, 10 g of methane, CH4, and 54 g of water, H2O, were heated in a


3

container of volume 4 dm . At equilibrium, 2.0 moles of hydrogen, H2, had formed. Write
an expression for the equilibrium constant, Kc, for the system, and use the data to
calculate a value for Kc, with units.
(8)

Sri Lankan School

235

(b)

The following table shows some data for enthalpies of formation, DHf.
Substance

DH f /kJ mol

CH4(g)

76

H2O(g)

242

CO2(g)

394

Use these data to calculate the enthalpy change for the reaction in (a).
(3)

(c)

In practice, the industrial production of hydrogen by this method is conducted at the


moderately high pressure of 30 atm, and the high temperature of 750 C, in the presence
of a nickel catalyst. Suggest why these conditions are used, considering the factors of rate
and yield.
(7)
(Total 18 marks)

106. This question is concerned with alkenes including ethene and buta-1,3-diene,
CH2 = CHCH = CH2.
(a)

(i)

The typical reactions of alkenes are addition reactions, for example their reactions
with bromine.
Explain why the reaction of ethene with bromine is described as an addition
reaction.
..........................................................................................................................
..........................................................................................................................
..........................................................................................................................
(1)

(ii)

Why should this reaction be carried out in the absence of sunlight or ultra-violet
radiation?
..........................................................................................................................
..........................................................................................................................
(1)

Sri Lankan School

236

Sri Lankan School

237

(iii)

Explain how, in this reaction, the bromine molecule is able to act as an electrophile,
even though it is normally non-polar.

(2)

(b)

(i)

When 1 mole of bromine molecules is added to 1 mole of buta-1,3-diene, the


principal product is 1,4-dibromobut-2-ene, CH2BrCH = CHCH2Br, a compound
which exists as two geometric isomers.
Draw the displayed formulae of both of these two isomers.

(2)

(ii)

State why geometric isomerism is possible in 1,4-dibromobut-2-ene.


[You may find it helpful to refer to the formulae you have drawn above].
..........................................................................................................................
..........................................................................................................................
..........................................................................................................................
(1)

Sri Lankan School

238

(c)

It has been suggested that cyclohexene, another alkene, could be made by a reaction
between ethene and buta-1,3-diene.
Using molecular formulae, the reaction could be represented by the equation
C2H4(g) + C4H6(g)

(i)

C6H10(l)

H = 200 kJ mol

Draw the displayed formula of cyclohexene.

(1)

(ii)

Decide whether high or low temperature and pressure would give the higher
proportion of cyclohexene at equilibrium. Justify your choice in each case.
Temperature .....................................................................................................
..........................................................................................................................
Pressure ............................................................................................................
..........................................................................................................................
..........................................................................................................................
(2)
(Total 10 marks)

Sri Lankan School

239

107. A saturated solution of calcium hydroxide, Ca(OH) 2(aq), has a pH of 9.6.


(a)

Write an expression linking hydrogen ion concentration and pH. Use this to calculate the
concentration of hydrogen ions in this solution.

(3)

(b)

(i)

The ionisation constant for water, Kw = 1.0 10

14

mol dm .

Write the expression for Kw.

Kw =

(1)

(ii)

Calculate the concentration of hydroxide ions in the saturated solution of calcium


hydroxide.

(1)

(iii)

Calculate the concentration of calcium hydroxide in the saturated solution.

(1)

Sri Lankan School

240

(iv)

Calculate the solubility of calcium hydroxide in g dm .


Give your answer to three significant figures.

(1)

(v)

Suggest why your calculated value may differ significantly from the value in
chemistry reference books.
.........................................................................................................................
.........................................................................................................................
(1)

(c)

An alternative method for finding the solubility of calcium hydroxide is to titrate 100 cm
3
of the saturated solution with hydrochloric acid of concentration 0.00100 mol dm .

Ca(OH)2 (aq) + 2HCl (aq) CaCl2 (aq) + 2H2O (l)


(i)

Calculate the pH of the hydrochloric acid.

(1)

(ii)

Use your answer to (b)(iii) and the information above to calculate the volume of
3
hydrochloric acid needed to neutralise 100 cm of the saturated calcium
hydroxide solution.

(3)

Sri Lankan School

241

(iii)

Sketch the titration curve for this reaction.

(2)

(iv)

Suggest why phenolphthalein is not a suitable indicator for this reaction.


..........................................................................................................................
..........................................................................................................................
(1)
(Total 15 marks)

108. In the Ostwald process for the production of nitric acid, ammonia is oxidised at 900 C over a
platinum/ rhodium alloy catalyst according to the equation:
4NH3(g) + 5O2(g)

4NO(g) + 6H2O(g)

The reaction is very exothermic.


(a)

(i)

Why do the concentrations of the substances in an equilibrium mixture remain


constant?
....
....
(1)

Sri Lankan School

242

(ii)

State with a reason the effect of an increase in pressure on this equilibrium system.
....
....
....
(2)

(b)

The mixture obtained from the catalyst chamber contains excess oxygen.
Write the equation for the further reaction that occurs on cooling this mixture.
....
(1)

(ii)

Show, by means of an equation, how the product in (b) (i) is used to make
nitric acid.
....
(2)

(c)

(i)

Sketch a Maxwell-Boltzmann distribution that could represent the energies of the


molecules in the Ostwald reaction system at a given temperature.

(2)

Sri Lankan School

243

(ii)

Use your diagram and any necessary explanation to show how the presence of a
catalyst leads to an increase in reaction rate at the same temperature.
....
....
....
....
....
....
(4)
(Total 12 marks)

109. (a)

Define the following terms.


(i)

pH .....................................................................................................................
(1)

(ii)

Kw ......................................................................................................................
(1)

(b)

Explain the meaning of the term strong, as applied to an acid or a base.


......
......
(1)

(c)

Calculate the pH of the following solutions.


(i)

HCl(aq) of concentration 0.200 mol dm .

(1)

Sri Lankan School

244

(ii)

NaOH (aq) of concentration 0.800 mol dm (Kw = 1.00 10

14

mol dm ).

(2)

(d)

HA is a weak acid with a dissociation constant Ka = 5.62 10 mol dm .


(i)

Write an expression for the dissociation constant, Ka, of HA.

(1)

(ii)

Calculate the pH of a 0.400 mol dm solution of HA.

(3)

Sri Lankan School

245

(e)

A buffer solution contains HA(aq) at a concentration of 0.300 mol dm , and its sodium
3
salt, NaA, at a concentration of 0.600 mol dm . Calculate the pH of this buffer solution.

(3)
(Total 13 marks)

110. Consider the following equation:


2SO2 + O2

2SO3
3

2.0 moles of SO2 and 1.0 mole of O2 were allowed to react in a vessel of volume 60 dm .
At equilibrium 1.8 moles of SO3 had formed and the pressure in the flask was 2 atm.
(a)

(i)

Write the expression for Kc for this reaction between SO2 and O2.

(1)

(ii)

Calculate the value of Kc, with units.

(3)

Sri Lankan School

246

(b)

The reaction between SO2 and O2 is exothermic. State the effect on the following, if the
experiment is repeated at a higher temperature:
(i)

Kc ..
(1)

(ii)

the equilibrium position ....................................................................................


(1)

(c)

State the effect of a catalyst on:


(i)

Kc ..
(1)

(ii)

the equilibrium position ....................................................................................


(1)

(d)

(i)

Write the expression for Kp for the reaction between SO2 and O2.

(1)

(ii)

Calculate the mole fractions of SO2, O2 and SO3 at equilibrium.

(2)

Sri Lankan School

247

(iii)

Calculate the partial pressures of SO2, O2 and SO3 at equilibrium.

(1)

(iv)

Calculate the value of Kp, with units.

(2)
(Total 14 marks)

111. (a)

The electronic configuration of a cobalt atom can be written as [Ar]3d 4s .


3+
Give the electronic configuratin of the Co ion.
......
(1)

(b)

(i)

By reference to the standard electrode potentials given below, suggest a reducing


3+
agent which might reduce aqueous Co ions to cobalt metal. Give your reasoning.
E /V
2+

Zn(s)

0.76

2+

Fe(s)

0.44

2+

Co(s)

0.28

2+

Sn(s)

0.14

Zn (aq) + 2e
Fe (aq) + 2e

Co (aq) + 2e
Sn (aq) + 2e
+

O2(g) + 2H (aq) + 2e
3+

Co (aq) + e

Sri Lankan School

2+

H2O2(aq)

Co (aq)

+0.68
+1.82

248

Sri Lankan School

249

Suitable reducing agent ....


Reasoning .....
.......
(3)

(ii)

Suggest two factors that might prevent a reducing agent from being as effective as
the electrode potentials might seem to suggest.
.......
.......
.......
.......
(2)

(c)

(i)

Write the formula of the hexaaquacobalt(II) ion.


.......
(1)

(ii)

Give an equation, involving the hexaaquacobalt(II)ion, to illustrate the process


of ligand exchange.
.......
(2)
(Total 9 marks)

Sri Lankan School

250

112. (a)

A mixture of hydrogen iodide, hydrogen and iodine (all in the gaseous state) establishes
dynamic equilibrium if a constant temperature is maintained.
2HI (g)
(i)

H2 (g) + I2 (g)

H = +9.6 kJ mol

Explain the meaning of the term dynamic equilibrium.


............................................................................................................................
............................................................................................................................
............................................................................................................................
(2)

(ii)

How, if at all, would the proportion of hydrogen iodide present at equilibrium


change if the temperature were to be increased? Justify your answer.
............................................................................................................................
............................................................................................................................
............................................................................................................................
............................................................................................................................
(2)

(iii)

The reaction is catalysed by metals such as gold and platinum. How, if at all, would
the proportion of hydrogen iodide present at equilibrium change if the reaction
were to be catalysed? Justify your answer.
............................................................................................................................
............................................................................................................................
(1)

Sri Lankan School

251

(b)

Part of an energy profile for this reaction is shown below. It is not intended to be to scale.

Complete the profile showing:

the products;

the progress of both uncatalysed and catalysed reactions;

labelled arrows to indicate the activation energies of both the uncatalysed and
catalysed reactions.
(4)
(Total 9 marks)

113. Thermochemical data, at 298 K, for the equilibrium between zinc carbonate, zinc oxide and
carbon dioxide is shown below.
ZnCO3(s)

ZnO(s) + CO2(g)

H = +71.0 kJ mol
1

S [ZnO(s)] =

+43.6 J mol K

S [ZnCO3(s)] =

+82.4 J mol K

S [CO2(g)] = +213.6 J mol K

Sri Lankan School

252

(a)

(i)

Suggest reasons for the differences between the three standard entropies.
............................................................................................................................
............................................................................................................................
............................................................................................................................
............................................................................................................................
(2)

(ii)

Calculate the entropy change for the system, S

system

, for this reaction. Include the

sign and units in your answer.

(2)

(b)

Calculate the entropy change for the surroundings, S


method clearly.

surroundings,

at 298 K, showing your

(2)

Sri Lankan School

253

(c)

(i)

Calculate the total entropy change for this reaction, S

total,

at 298 K.

(1)

(ii)

What does the result of your calculation in (c)(i) indicate about the natural
direction of this reaction at 298 K?
Justify your answer.
............................................................................................................................
............................................................................................................................
(1)

(d)

(i)

Write an expression for the equilibrium constant, Kp, for this reaction.

(1)

Sri Lankan School

254

(ii)

State how you would alter ONE condition to increase the yield of carbon dioxide
from this equilibrium reaction.
Justify your answer.
...........................................................................................................................
...........................................................................................................................
...........................................................................................................................
...........................................................................................................................
(2)
(Total 11 marks)

114. The reaction in the Haber Process that is used to produce ammonia is a homogeneous dynamic
equilibrium:
N2(g) + 3H2(g)

(a)

2NH3(g)

DH = 92 kJ mol

State the meaning of the terms:


(i)

dynamic equilibrium;
............................................................................................................................
............................................................................................................................
(2)

(ii)

homogeneous.
............................................................................................................................
(1)

(b)

Give, with a reason in each case, the effect of the following on the position of the
equilibrium above:

Sri Lankan School

255

(i)

an increase in pressure;
............................................................................................................................
............................................................................................................................
............................................................................................................................
(2)

(ii)

an increase in temperature.
............................................................................................................................
............................................................................................................................
............................................................................................................................
(2)
(Total 7 marks)

115. (a)

Methane reacts with steam in a reversible reaction. In industry this reaction, carried out at
a pressure of 30 atm, is used to produce hydrogen for the manufacture of ammonia
CH4(g) + H2O(g)

(i)

CO(g) + 3H2(g)

DH = +210 kJ mol

Define the term partial pressure as applied to a gas mixture.


............................................................................................................................
............................................................................................................................
(1)

(ii)

Sri Lankan School

Write an expression for the equilibrium constant, Kp, for this reaction.

256

(1)

Sri Lankan School

257

(iii)

State and explain the effect of increasing the total pressure on the position of this
equilibrium;
............................................................................................................................
............................................................................................................................
............................................................................................................................
............................................................................................................................
............................................................................................................................
(2)

(b)

State the effect on the value of Kp for this equilibrium of the following.
(i)

Increasing the total pressure.


............................................................................................................................
(1)

(ii)

Increasing the temperature.


............................................................................................................................
(1)

(iii)

Adding a catalyst.
............................................................................................................................
(1)

(c)

There is a theory that methane, CH4, constantly leaks from the earths crust. This is not
noticeable on land but at the bottom of a cold sea, such as off the Canadian coast, the
methane is trapped in a solid cage of water molecules.
CH4(g) + 6H2O(s)

Sri Lankan School

[CH4(H2O)6](s)
methane hydrate

258

At 29 C the equilibrium pressure of the methane is 101.3 kPa.


(i)

Write an expression for Kp for this equilibrium.

(1)

(ii)

Deduce the value of Kp at 29 C, stating its units.

(1)

(iii)

At 0 C the equilibrium pressure of methane rises to 2600 kPa. What does this tell
you about the effect of temperature change on the position of equilibrium and
about the enthalpy change for this reaction?
............................................................................................................................
............................................................................................................................
............................................................................................................................
(2)

Sri Lankan School

259

(iv)

Some people have suggested collecting the methane hydrate from the bottom of the
sea and allowing it to warm up to 0 C on board a ship. Comment on whether this
would be a useful method for collecting methane.
............................................................................................................................
............................................................................................................................
............................................................................................................................
(1)
(Total 12 marks)

116. Ethanoic acid, CH3COOH, is a weak acid which can be used, with its salts, to make buffer
solutions.
(a)

Explain what is meant by the term weak acid.


....................................................................................................................................
....................................................................................................................................
(1)

(b)

Explain what is meant by the term buffer solution.


....................................................................................................................................
....................................................................................................................................
(2)

Sri Lankan School

260

(c)

An aqueous solution of ethanoic acid of concentration 1.00 mol dm has a pH of 2.8.


3
3
Sketch, with care, how the pH changes during the titration of 25.0 cm 1.00 mol dm
aqueous ethanoic acid with aqueous sodium hydroxide of the same concentration.

14
pH
12
10
8
6
4
2
0

10

20

30
40
50
V o lu m e o f a lk a li a d d e d /c m 3
(4)

(d)

Indicate on your sketch the portion of the curve where the mixture is behaving as a buffer.
(1)

(e)

(i)

Write an expression for the dissociation constant, Ka, for ethanoic acid.

(1)

(ii)

Explain how the pKa of ethanoic could be found from the graph.
............................................................................................................................
............................................................................................................................
(2)
(Total 11 marks)

Sri Lankan School

261

Sri Lankan School

262

117. (a)

The bombardier beetle Metrius contractus persuades potential predators to disappear by


firing a boiling mixture of irritants at them. The reaction producing this ammunition is a
redox reaction, H2O2 being the oxidising agent.
The two half-reactions involved are:

O H

E /V

+ 2H

O H

+ 2 e + 0 .7 0

O
H 2O

(i)

+ 2H

+ 2e

2H 2O

+ 1 .7 7

Write the overall equation for the reaction and show that the reaction is feasible.
(3)

(ii)

The beetle makes use of an enzyme catalyst in the reaction. Explain in general
terms how catalysts increase the rate of a chemical reaction using a graph of the
Maxwell-Boltzmann distribution of molecular energies.
(5)

(iii)

The reaction is highly exothermic; in principle its enthalpy of reaction could be


found by using average bond enthalpies. By a consideration of the structure and
bonding in the compounds involved, suggest why the use of the average bond
enthalpies for C==O, C C, C==C and O H would give a highly inaccurate
answer for the enthalpy of reaction.
(2)

(b)

On heating hydrogen peroxide decomposes according to the equation


2H2O2 2H2O + O2
Hydrogen peroxide is marketed as an aqueous solution of a given volume strength. The
3
3
common 20-volume solution gives 20 dm of oxygen from 1 dm of solution. What is the
3
concentration in g dm of such a solution? (Molar volume of any gas at the temperature
3
and pressure of the experiment is 24 dm .)
(3)

Sri Lankan School

263

Sri Lankan School

264

(c)

Hydrogen peroxide, H2O2, can also act as a reducing agent.


The rapid oxidation of hydrogen peroxide was used in World War II to generate steam to
launch the V1 flying bomb. H2O2 (100 volume) was reacted with acidified potassium
manganate(VII) solution.

(i)

Write the half-equation for the oxidation of hydrogen peroxide to oxygen, O 2.


(1)

(ii)

2+

The MnO 4 ions are reduced to Mn during the reaction. Derive the overall
equation for the reaction between H2O2 and acidified KMnO4.
(2)

(iii)

Suggest in terms of the collision theory of chemical kinetics why 100-volume


3
3
hydrogen peroxide (this gives l00 dm of oxygen from 1 dm of hydrogen peroxide
when it decomposes to water and oxygen) was used rather than the more common
20-volume solution.
(2)
(Total 18 marks)

118. (a)

The first stage in the manufacture of sulphuric acid is the Contact Process.
2SO2 + O2

2SO3
4

At 400 C the equilibrium constant Kp = 3.00 10 atm . A catalyst of vanadium(V)


oxide is used. In a particular equilibrium mixture at 400 C the partial pressures of
sulphur dioxide and of oxygen were 0.100 atm and 0.500 atm respectively. Show that the
yield of SO3 is about 95% of the equilibrium mixture.
(5)

Sri Lankan School

265

(b)

(i)

Pure sulphuric acid is a viscous liquid with a high boiling temperature of


338 C. It has the structure:
O
S
O

O H
O H

Suggest in terms of the intermolecular forces in sulphuric acid why it has such a
high boiling temperature.
(3)

(ii)

Sulphuric acid dissolves in water in a highly exothermic reaction


May her rest be long and placid,
She added water to the acid;
The other girl did what we taught her,
And added acid to the water.
Suggest why sulphuric acid must always be added to water to dilute it rather than
the other way round.
(2)

(c)

Sulphuric acid dissociates in water according to the equations:

H2SO4 + H2O H3O + HSO 4

HSO 4 + H2O

H3O + SO 4

The dissociation constant for the first dissociation is very large; that for the second is 0.01
3
mol dm at 25 C.

(i)

Calculate the pH of an aqueous solution containing 0.200 mol dm hydrogen ions.


(1)

(ii)

The pH of 0.100 mol dm sulphuric acid is 0.98. Explain why this is so close to
3
the pH of 0.100 mol dm HCl which is 1.0.
(3)

Sri Lankan School

266

Sri Lankan School

267

(d)

Sulphuric acid is used as the electrolyte in the lead-acid battery found in cars. The
electrodes are made from lead and from lead(IV) oxide. As the cell discharges, the lead
and the lead(IV) oxide are both converted to lead(II) sulphate, and the sulphuric acid
concentration falls.
(i)

Use the information above to deduce the two half equations occurring in the lead
acid battery.
(3)

(ii)

Hence write an equation to represent the overall process taking place as the cell
discharges.
(1)
(Total 18 marks)

119. Ammonia reacts with oxygen, in a reversible, exothermic reaction, as shown in the equation
below. This is the first stage in the manufacture of nitric acid.
4NH3(g) + 5O2(g)

(a)

(i)

4NO(g) + 6H2O(g)

State and explain the effect of an increase in pressure on the position of this
equilibrium.
..........................................................................................................................
..........................................................................................................................
..........................................................................................................................
(2)

(ii)

State and explain the effect of an increase in temperature on the position of the
equilibrium.
..........................................................................................................................
..........................................................................................................................
..........................................................................................................................
(2)

Sri Lankan School

268

(b)

(i)

State and explain the effect of an increase in pressure on the rate of the reaction.
..........................................................................................................................
..........................................................................................................................
..........................................................................................................................
..........................................................................................................................
(3)

(ii)

State and explain the effect of an increase in temperature on the rate of the
reaction.
..........................................................................................................................
..........................................................................................................................
..........................................................................................................................
..........................................................................................................................
(3)

(c)

(i)

Name the catalyst used in the reaction, during the manufacture of nitric acid.
..........................................................................................................................
(1)

(ii)

Explain the effect of a catalyst on the rate of the reaction.


..........................................................................................................................
..........................................................................................................................
..........................................................................................................................
..........................................................................................................................
(3)

Sri Lankan School

269

(iii)

Suggest why the catalyst is in the form of a gauze or mesh.


..........................................................................................................................
..........................................................................................................................
(1)
(Total 15 marks)

120. (a)

What is meant by the term weak acid?


....................................................................................................................................
....................................................................................................................................
....................................................................................................................................
(2)

(b)

A weak acid, represented by HA, dissociates in water according to the equation:

HA(aq) + H2O(l)

H3O+(aq) + A(aq)

Write an expression for the dissociation constant, Ka, for HA.

(1)

Sri Lankan School

270

(c)

25 cm of 1.00 mol dm aqueous HA, was titrated with 1.00 mol dm aqueous sodium
hydroxide and the pH measured throughout. The titration curve is shown below.

14
12
pH

10
8
6
4
2
0

10

20
30
40
V o lu m e o f s o d iu m h y d ro x id e a d d e d / c m

50
3

Use the titration curve to find:


(i)

the value of the pH at the end point of the titration.


............................................................................................................................
(1)

(ii)

the pH of an aqueous solution of the salt NaA.


............................................................................................................................
(1)

(iii)

the value of pKa for the acid HA and, hence the value Ka.
pKa ...................................................................................................................
Ka .....................................................................................................................
(2)

Sri Lankan School

271

(d)

Some of the solutions made during this titration would act as buffer solutions.
(i)

What is meant by the term buffer solution?


............................................................................................................................
............................................................................................................................
............................................................................................................................
(2)

(ii)

Use the titration curve to find:


the range of pH values over which this mixture acts as a buffer;
from ....................................................

to ....................................................
(1)

the pH of the most efficient buffer solution.


............................................................................................................................
(1)

(e)

Suggest, with reasoning, whether methyl orange or phenolphthalein would be the better
indicator for this titration.
Choice ........................................................................................................................
Reasoning ..................................................................................................................
....................................................................................................................................
....................................................................................................................................
(3)

(f)

Explain why, as the titration proceeds, the flask becomes warm but not as warm as it
3
would in a similar titration using 1.00 mol dm solutions of hydrochloric acid and
sodium hydroxide.
....................................................................................................................................
....................................................................................................................................
....................................................................................................................................
(3)

Sri Lankan School

272

Sri Lankan School

273

(g)

A different monobasic weak acid has a dissociation constant of 1.8 10


(i)

mol dm .

Define pH.
............................................................................................................................
(1)

(ii)

Calculate the pH of a 1.00 mol dm aqueous solution of this acid.

(3)
(Total 21 marks)

121. In the vapour phase sulphur trioxide dissociates:


2SO3(g)

(a)

(i)

2SO2(g) + O2(g)

Write an expression for Kp for this dissociation.

(1)

Sri Lankan School

274

(ii)

At a particular temperature, 75% of the sulphur trioxide is dissociated, producing a


pressure of 10 atm. Calculate the value of Kp at this temperature paying, attention
to its units.

(5)

(b)

Solid vanadium(V) oxide, V2O5, is an effective catalyst for this reaction. State the effect
of using double the mass of catalyst on:
(i)

the position of the equilibrium;


............................................................................................................................
............................................................................................................................
(1)

(ii)

the value of Kp.


............................................................................................................................
............................................................................................................................
(1)
(Total 8 marks)

122. (a)

The acid dissociation constant, Ka for ethanoic acid is 1.80 10 mol dm .


(i)

Write an equation for the dissociation of aqueous ethanoic acid.


......................................................................................................................
(1)

Sri Lankan School

275

(ii)

Give the expression for the acid dissociation constant, Ka, for aqueous ethanoic
acid.

(1)

(iii)

Calculate the pH of 2.00 mol dm ethanoic acid.

(3)

(b)

When ethanoic acid and propan-1-ol are heated together in the presence of a little
concentrated sulphuric acid an ester is produced.
(i)

Draw the structural formula of the ester produced in this reaction.

(2)

(ii)

Give the name of the ester.


......................................................................................................................
(1)

(iii)

State the function of the sulphuric acid.


......................................................................................................................
(1)

Sri Lankan School

276

Sri Lankan School

277

(c)

6.00g of ethanoic acid was mixed with 6.00g of propan-1-ol and 0.00100mol of
concentrated sulphuric acid was added. The mixture was heated under reflux until no
further reaction was detectable. The mixture was then cooled rapidly to room temperature
3
3

and titrated with 1.00 mol dm sodium hydroxide solution. 35.0 cm of the 1.00 mol dm
3
sodium hydroxide solution was required for complete neutralisation.
(i)

Calculate the volume of sodium hydroxide solution that would have been needed if
only the 0.00100mol of concentrated sulphuric acid had been in the flask.

(2)

(ii)

Calculate the volume of sodium hydroxide solution that must have been needed to
react with the ethanoic acid remaining in the cooled mixture.

(1)

(iii)

Calculate the amount (in moles) of ethanoic acid remaining.

(1)

Sri Lankan School

278

(iv)

Hence, calculate the amount (in moles) of propan-1-ol, ester and water in the final
cooled mixture.

(3)

(v)

Calculate the value of the equilibrium constant under these conditions. The
3
volume of the reaction mixture should be taken as 100 cm .

(2)

(d)

The reverse of esterification is hydrolysis. In experiments to study the hydrolysis of this


ester with aqueous sodium hydroxide, the following data were obtained.
Concentration of
ester
3
/mol dm

Concentration of
sodium
hydroxide
3
/mol dm

0.1

1.0

6 10

0.1

2.0

1.2 10

0.2

2.0

2.4 10

Experiment

Sri Lankan School

Rate of
reaction

/mol dm
3 1
s
4

279

(i)

Determine the order of the reaction with respect to the:


ester;..............................................................................................................
(1)

sodium hydroxide...........................................................................................
(1)

(ii)

Calculate the value of the rate constant, k, for this reaction including its units.

(3)
(Total 23 marks)

123. This question is about Group 7 of the Periodic Table - the halogens.
The standard electrode potentials for these elements are given below.
Electrode reaction
1
2

1
2

1
2

1
2

Sri Lankan School

F2 + e

Cl2 + e

I2 + e

+1.36

+1.07

Cl

Br
I

/V

+2.87

Br2 + e

+0.54

280

(a)

(i)

Define the term standard electrode potential.


......................................................................................................................
......................................................................................................................
......................................................................................................................
(2)

(ii)

State which element or ion in the table above is the strongest oxidising agent.
......................................................................................................................
(1)

(b)

The standard electrode potentials for chromium(III) changing to chromium(II) and for
chromium(VI) changing to chromium(III) are given below.
Cr
1
2

3+

+e

Cr2 O7 + 7H + 3e

(i)

Cr

2+

Cr

3+

7
2

H2O

= 0.41V

= +1.33V

On the basis of the data provided, list those halogens which will oxidise
chromium(II) to chromium(III).
......................................................................................................................
(1)

(ii)

On the basis of the data provided, list those halogens which will oxidise
chromium(II) to chromium(III) but not to chromium(VI).
......................................................................................................................
(1)

Sri Lankan School

281

(iii)

Chromium(II) in aqueous solution is sky blue while aqueous chromium(III)


solution is dark green. Describe how you would show that your prediction in part
(ii) actually worked in practice.
......................................................................................................................
......................................................................................................................
......................................................................................................................
......................................................................................................................
(2)

124. (a)

When solid sodium nitrate is heated with concentrated sulphuric acid the reaction shown
below occurs:

(i)

NaNO3 + H2SO4

HNO3 + NaHSO4

..........

..........

..........

..........

Identify the two conjugate acid-base pairs in this reaction. Write your answer in
the spaces below the equation.
(2)

(ii)

The nitric acid produced is evolved as a gas under these conditions. Explain the
effect of this on the position of equilibrium on heating the mixture in an open
container.
......................................................................................................................
......................................................................................................................
(2)

(iii)

Nitric acid molecules in the vapour phase can be represented as


O

O H

Sri Lankan School

282

Draw a diagram of the likely shape of this molecule and justify your prediction.

......................................................................................................................
......................................................................................................................
......................................................................................................................
(3)

(b)

Dilute sulphuric acid is a strong acid but very concentrated sulphuric acid is weak acid.
Explain the meanings of the terms weak and dilute as applied to acids.
Weak.......................................................................................................................
................................................................................................................................
Dilute.......................................................................................................................
................................................................................................................................
(2)

(c)

Heating solid sodium nitrate, NaNO3, to decomposition requires a temperature of about


550C and produces oxygen as the only gaseous product. Solid magnesium nitrate,
Mg(NO3)2, decomposes on heating at a much lower temperature.
(i)

Write an equation for the decomposition of sodium nitrate.


......................................................................................................................
(1)

Sri Lankan School

283

(ii)

Relate the ease of decomposition of the two nitrates to the relative sizes and
charges of the cations.
......................................................................................................................
......................................................................................................................
......................................................................................................................
......................................................................................................................
......................................................................................................................
(3)
(Total 13 marks)

125. The reaction between sulphur dioxide and oxygen is a dynamic equilibrium.
2SO2 + O2
(a)

2SO3

DH = 196 kJmol

Explain what is meant by dynamic equilibrium.


..................................................................................................................................
..................................................................................................................................
..................................................................................................................................
(2)

(b)

In the table below state the effect on this reaction of increasing the temperature and of
increasing the pressure.
Effect on the rate of
the
reaction

Increasing the
temperature

Effect on the position


of
equilibrium

Increases

Increasing the pressure

(3)
Sri Lankan School

284

(c)

This reaction is one of the steps in the industrial production of sulphuric acid. The
normal operating conditions are a temperature of 450 C, a pressure of 2 atmospheres
and the use of a catalyst.
Justify the use of these conditions.
(i)

A temperature of 450 C:
......................................................................................................................
......................................................................................................................
......................................................................................................................
(3)

(ii)

A pressure of 2 atmospheres:
......................................................................................................................
......................................................................................................................
......................................................................................................................
(2)

(iii)

A catalyst:
......................................................................................................................
......................................................................................................................
......................................................................................................................
(1)

(d)

Give the name of the catalyst used.


..................................................................................................................................
(1)

(e)

Give one large scale use of sulphuric acid.


..................................................................................................................................
(1)
(Total 13 marks)

Sri Lankan School

285

Sri Lankan School

286

126. (a)

(i)

Calculate the concentration, in mol dm , of a solution of hydrochloric acid, HCl,


which has a pH of 1.13.

(1)

(ii)

Calculate the concentration, in mol dm , of a solution of chloric(l) acid, HOCl,


which has a pH of 4.23.
8
3
Chloric(l) acid is a weak acid with Ka = 3.72 10 mol dm .

(4)

(b)

The pH of 0.100 mol dm sulphuric acid is 0.98.


(i)

Calculate the concentration of hydrogen ions, H , in this solution.

(1)

(ii)

Write equations to show the two successive ionisations of sulphuric acid, H 2SO4, in
water.
......................................................................................................................
......................................................................................................................
(2)

Sri Lankan School

287

Sri Lankan School

288

(iii)

Suggest why the concentration of hydrogen ions is not 0.20 mol dm in 0.100 mol
3
dm sulphuric acid.
......................................................................................................................
......................................................................................................................
(1)

(c)

Many industrial organic reactions produce hydrogen chloride as an additional product.


This can be oxidised to chlorine by the Deacon process:
4HCl(g) + O2(g)

2Cl2(g) + 2H2O(g)

DH = 115 kJ mol .

0.800 mol of hydrogen chloride was mixed with 0.200 mol of oxygen in a vessel of
3
volume 10.0 dm in the presence of a copper(I) chloride catalyst at 400 C. At
equilibrium it was found that the mixture contained 0.200 mol of hydrogen chloride.
(i)

Write an expression for the equilibrium constant Kc.

(1)

(ii)

Calculate the value of Kc at 400 C.

(4)

Sri Lankan School

289

(d)

State and explain the effect, if any, on the position of equilibrium in (c) of:
(i)

decreasing the temperature;


......................................................................................................................
......................................................................................................................
......................................................................................................................
(2)

(ii)

decreasing the volume;


......................................................................................................................
......................................................................................................................
......................................................................................................................
(2)

(iii)

removing the catalyst.


......................................................................................................................
......................................................................................................................
......................................................................................................................
(2)
(Total 20 marks)

127. (a)

(i)

Draw dot and cross diagrams to show the electronic structure of the ammonia and
of the boron trifluoride molecules. Hence deduce their shapes and suggest values
for the HNH and FBF bond angles in these molecules.
(5)

Sri Lankan School

290

(ii)

Explain, in terms of the intermolecular forces involved, the variation of the boiling
temperatures of the Group 5 hydrides listed below.
Hydride

Boiling Temperature/ K

Ammonia, NH3

240

Phosphine, PH3

183

Arsine, AsH3

218

Stibine, SbH3

256
(5)

(b)

When ammonia and boron trifluoride are mixed, an addition compound, H3NBF3, is
formed.
(i)

Suggest how the nitrogen-boron bond forms between the two molecules in the
addition compound.
(1)

(ii)

Suggest how the HNH and the FBF bond angles would change when the addition
compound forms.
(2)

(c)

One of the early rocket fuels was hydrazine. It burns in oxygen as follows:
H2NNH2(g) + O2(g) N2(g) + 2H2O(g)
4

When 1.00 kg of hydrazine is burnt in excess oxygen 1.83 10 kJ of heat energy is


released. Use this and the average bond enthalpies below to calculate the NN bond
enthalpy.

Bond

Bond
enthal
py
/kJ
1
mol

Bon
d

Bond
enthalp
y
/kJ
1
mol

NH

+388

O==O

+496

NN

+944

HO

+463
(4)
(Total 17 marks)

Sri Lankan School

291

Sri Lankan School

292

128. (a)

Benzocaine, C9H11O2N, is an aromatic compound which is used commercially in creams


to alleviate sunburn.
+

Benzocaine reacts with dilute acids to form the ion C 9H12O2N and with ethanoyl
chloride to form C11H13O3N.
When benzocaine is heated under reflux with aqueous sodium hydroxide and the
solution obtained is neutralised, two compounds X and Y are formed.
X has a formula of C7H7O2N and is a solid with a melting temperature of 190 C. It is
soluble in water.
Y is a volatile liquid with a formula C2H6O which gives steamy fumes with
phosphorus pentachloride.
X reacts with sodium hydrogencarbonate solution to give a gas which turns lime water
milky. It also reacts with a solution of sodium nitrite and hydrochloric acid between 0
C and 5 C to produce a substance which reacts with phenol to give an orange
precipitate, Z.

These reactions are summarised as follows.

C 9H

12O 2

H + (a q )

C 9 H 11 O 2 N
B e n z o c a in e

C H 3C O Cl

11

13O 3

1 . N a O H ( a q ) h e a t/r e f lu x
2 . H C l(a q ) u n til n e u tra l
C 7H 7O 2N
X
N a H C O 3 (a q )

C 7H 6O 2N N a

(i)

C 2 H 6O
Y

1 . H N O 2 b e tw e e n 0 C a n d 5 C
2. phenol
o ra n g e p p t. Z

Deduce a structural formula for benzocaine and explain its three reactions shown
above. You may either describe the types of reaction or write the equations for the
reactions.
(6)

Sri Lankan School

293

(ii)

Write equations for the two reactions of X. Include in your answer the structural
formula of Z.
(3)

(iii)

Explain why substance X has a fairly high melting temperature and why it is
soluble in water.
(3)

(b)

Substance X is a weak monobasic acid and for the purpose of the remainder of this
question you may write its formula as HA.
X has a relative molecular mass of 137, with a pKa value of 4.92 at 25 C.
(i)

Calculate the pH of a solution containing 21.37g of X per dm at a temperature of


25 C.
(4)

(ii)

50.0 cm of this solution was mixed with 50.0 cm of a 0.100 mol dm solution of
sodium hydroxide. Calculate the concentration of the salt of X produced, and the
concentration of the acid X left unreacted.
Hence calculate the pH of the mixed solution.
(4)

(c)

The standard enthalpy change at 25 C for the neutralisation of a strong acid by a strong
1
base is 57.2 kJ mol .
The standard enthalpy change for the ionisation of the weak acid HA in water is
1
+8.3 kJ mol .
(i)

Write the ionic equation for the neutralisation of a strong acid by a strong base and
hence calculate the standard enthalpy of neutralisation of the acid HA.
(3)

(ii)

State and explain how the value of Ka of the acid X and hence the pH of the
solution in (b)(i) would change if the temperature of the solution were increased.
(2)
(Total 25 marks)

Sri Lankan School

294

129. (a)

Propan-1ol, C2H5CH2OH can be oxidised to propanoic acid, C2H5COOH.


(i)

State the names of the reagents necessary for this conversion.


............................................................................................................................
............................................................................................................................
(2)

(ii)

5.67 of propan-1-o1 was oxidised in a reaction with a 64% yield. Calculate the
mass of propanoic acid produced.

(3)

(b)

Propanoic acid is a weak acid. It ionises according to the equation:


C2H5COOH(aq)+H2O(l)

(i)

C2H5COO (aq) + H3O (aq)

DH

= +6kJ mol

State and explain the effect on the position of this equilibrium of:
an increase in temperature;
............................................................................................................................
............................................................................................................................
(2)

an addition of solid sodium propanoate.


............................................................................................................................
............................................................................................................................
(2)

(ii)

How does the addition of sodium propanoate affect the pH of the solution of
propanoic acid?
............................................................................................................................

Sri Lankan School

295

(1)

Sri Lankan School

296

(c)

(i)

Sketch, with reasonable accuracy, how the pH changes during the titration of
3
25cm of a weak acid, such as propanoic acid, with sodium hydroxide solution of
the same concentration.

14
12
10
pH

8
6
4
2
0

10

20

30

40

50

v o lu m e N a O H /c m

(ii)

The table contains some data about three indicators.


Indicator

pKind

Acid colour

Alkaline
colour

Bromophenol blue

4.0

yellow

blue

Bromothymol blue

7.0

yellow

blue

Thymol blue

8.9

yellow

blue

State which of these indicators would be best for this titration. Give a reason for
your choice.
Indicator ...........................................................................................................
Reason ..............................................................................................................
...........................................................................................................................
(2)

Sri Lankan School

297

(d)

The standard enthalpy change of neutralisation of some acids with sodium hydroxide is
tabulated below:
Acid

(i)

DH /kJ
1
mol

Propanoic acid, C2H5COOH

51

Hydrocyanic acid, HCN

12

Hydrochloric acid, HCl

57

Nitric acid, HNO3

57

Why are the values for the enthalpy change of neutralisation of the two strong
acids the same?
..........................................................................................................................
..........................................................................................................................
(1)

(ii)

Why is the enthalpy change of neutralisation of hydrocyanic acid so much less than
that of hydrochloric acid?
..........................................................................................................................
..........................................................................................................................
(2)
(Total 19 marks)

130. (a)

When solid calcium nitrate is heated, brown fumes of nitrogen dioxide, NO 2, are seen and
the solid remaining after decomposition is calcium oxide.
(i)

Write a balanced equation for the thermal decomposition of calcium nitrate.


.........................................................................................................................
(2)

Sri Lankan School

298

(ii)

Describe the changes you would see when cold water is added drop by drop to cold
calcium oxide and give the chemical equation for the reaction.
.........................................................................................................................
.........................................................................................................................
.........................................................................................................................
.........................................................................................................................
(3)

(iii)

State whether barium nitrate will decompose more easily or less easily than
calcium nitrate on heating with a Bunsen burner.
.........................................................................................................................
(1)

(iv)

Account for the trend in the thermal stability of the nitrates of the elements in
group 2.
.........................................................................................................................
.........................................................................................................................
.........................................................................................................................
.........................................................................................................................
(3)

(b)

The brown fumes in part (a) are not pure NO 2 but a mixture of N2O4 and NO2.
N2O4(g)
Pale yellow

2NO2(g)
dark brown

A transparent glass syringe was filled with the gaseous mixture of N 2O4 and NO2 and its
tip sealed. When the piston of the syringe was rapidly pushed well into the body of the
syringe, thereby compressing the gas mixture considerably, the colour of the gas became
momentarily darker but them became lighter again.

(i)

Suggest why compressing the gases causes the mixture to darken.


.........................................................................................................................

Sri Lankan School

299

.........................................................................................................................
(1)

Sri Lankan School

300

(ii)

Explain why the mixture turns lighter on standing.


.........................................................................................................................
.........................................................................................................................
.........................................................................................................................
(2)

(iii)

Write an expression for the equilibrium constant, Kp, for this equilibrium.

(1)

(iv)

1.0 mole of N2O4 was allowed to reach equilibrium at 400K. At equilibrium the
partial pressure of N2O4 was found to be 0.15 atm.
Given that the equilibrium constant Kp for this reaction is 48 atm, calculate the
partial pressure of NO2 in the equilibrium mixture.

(3)
(Total 16 marks)

131. Sorbic acid has the formula


CH3CH=CHCH=CHCOOH
Together with its potassium salt sorbic acid is used in foodstuffs as an inhibitor of fungal and
mould growth.

(a)

Sorbic acid can show geometric isomerism.


(i)

Explain carefully why sorbic acid shows geometric isomerism.


(2)

(ii)

Sri Lankan School

State, with a reason, the number of geometric isomers that sorbic acid has.

301

(2)

Sri Lankan School

302

(b)

Sorbic acid and potassium sorbate together form a buffer system.


(i)

What is a buffer system?


(2)

(ii)

Write the chemical equations that are relevant to the functioning of the mixture as a
buffer, and use them to explain how buffering action arises. You may represent
sorbic acid as RCOOH and potassium sorbate as RCOOK.
(5)

(c)

Sorbic acid will react with a solution of bromine in tetrachloromethane.


(i)

Write an equation for the complete reaction of sorbic acid with bromine.
(1)

(ii)

A 7.00g sample of sorbic acid was reacted with a solution of bromine in


3
tetrachloromethane; 125cm of the solution was required. Calculate the
3
concentration of the bromine solution in mod dm .
(4)

(d)

The bromine-containing product from (c)(i) can be reacted with sodium hydroxide in
aqueous ethanol to give a substitution reaction.
(i)

Explain why aqueous ethanol is used as the solvent rather than, say, ethanol alone.
(2)

(ii)

Write the structural formula of the product.


(1)

(e)

Potassium sorbate can function as a soap.


(i)

Explain why this is so.


(1)

(ii)

Soaps are in general made by the alkaline hydrolysis of fats. Write an equation to
illustrate this reaction.
(3)
(Total 25 marks)

Sri Lankan School

303

Вам также может понравиться